Download as pdf or txt
Download as pdf or txt
You are on page 1of 104

NSEJS-2016 (IJSO STAGE-I)

Date of Examination : 20th November, 2016


PAPER CODE - JS531

SOLUTIONS
1. Rod AB of radius 2r is joined with rod BC of radius r. They are of same material and are of same
length. The combination carries a current I. Choose the correct statement.
A B C

I I

(a) VAB = 4VBC (b) Current per unit in AB and BC are equal
(c) Resistance of AB is greater than of BC (d) VBC = 4VAB
Ans. (d)

VAB IR AB Irl / p(2 p)2 1


Sol. VBC = IR BC = Irl / pr 2 = 4
or VBC = 4VAB
2. The statement “a is not less than 4” is correctly represented by
(a) a < 4 (b) a > 4 (c) a ³ 4 (d) a £ 4
Ans.
Sol. "a is not less than 4"
Þ a is greater or equal to 4
Þ a³4
So option (c) is correct
3. A chemist mixes two ideal liquids A and B to form a homogeneous mixture. The densities of the
liquids are 2.0 g/mL for A and 3 g/mL for B. When she drops a small object into the mixture, she finds
that the object becomes suspended in the liquid that is it neither sinks to the bottom nor does it float on
the surface. If the mixture is made of 40% A and 60% B, by volume, what is the density of the object?
(a) 2.60 g/mL (b) 2.50 g/mL (c) 2.40 g/mL (d) 1.50 g/mL
Ans. (a)
Sol. density of mixture = density of object
VA r A + VBrB 0.4 ´ 2.0 + 0.6 ´ 3
= =
VA + VB 1
= 0.8 + 1.8 = 2.6 g/mL
4. How many different compounds can have the formula C3 H4 ?
(a) One (b) Two (c) Three (d) Four
Ans. (c) Three
Sol. Structures are : CH3 – C º CH

CH2 = C = CH2 and

1
5. In the figure shown, the current carrying loop is fixed, where as current carrying straight conductor is
free to move. Then straight wire will (ignore gravity)
(a) remain stationary
(b) move towards the loop I1 I2

(c) move away from the loop


(d) rotate about the axis perpendicular to plane of paper
Ans. (b)
Sol. Due to current carrying loop, magnetic field on straight wire will be in outward direction so a force
towards right will be exerted on this wire.
6. Two friends A and B watched a car from the top of their buildings. Angle of depression for A was 10°
more than angle of depression for B, then
(a) A's apartment is taller than B's apartment
(b) B's apartment is taller than A's apartment
(c) A's apartment and B's apartment have same height
(d) We cannot compare the heights of the two apartments
Ans. (d)
Sol. Let 'H1' and 'H2' i.e. the height of two buildings, then A D

H2 H1
tan q = , tan (q + 10°) = H1 H2
b a

Þ H2 = b tan q, q + 10° q
B C
H1 = a tan(q + 10°) a E b
Now, we can compare the height of two building only when a and b are known.
so option (d) is correct
7. How many times would a red blood cell pass through the heart during one complete cycle?
(a) Once (b) Twice (c) 4 times (d) 72 times
Ans. (b)
Sol. In double circulation, blood passes twice from heart.
8. A gene has two alleles P(dominant) and p(recessive). The homozygous recessive combination leads
to death in the embryo stage. If two individuals with genotype Pp are mated, out of the offspring that
survive to adulthood, what is the probability of the genotype to be Pp?
(a) 0.75 (b) 0.33 (c) 0.5 (d) 0.67
Ans. (d)
Sol. Pp × Pp

pp Pp Pp pp

2
As mention in question homozygous recessive will die. So pp will die then Pp will be = 0.67.
3

2
1
9. A convex mirror of focal length f produces an image of size equal to times the size of the object.
n
Then the object distance is

f
(a) nf (b) (c) (n + 1)f (d) (n – 1)f
n
Ans. (d)
v 1 u
Sol. = , v=
u n n
Writing mirror formula,
1 1 1
+ =
v u f
n 1 1
- =
u u f

1 1
or, (n – 1) =
u f
\ u = (n – 1)f

10. Total surface area of a sphere S with radius 2 + 3 cm is

(a) 400p (5 + 2 6) sqmm (b) p ( 2 + 3)2 sqcm

(c) 2p ( 2 + 3)2 sqcm (d) 40p (5 + 2 6) sqmm


Ans. (a)

Sol. r = 2+ 3

S.A of sphere = 4p ( 2 + 3 )2

= 4p (5 + 2 6 ) cm2

= 400 p (5 + 2 6 ) mm2
So option (a) is correct.
11. There are many elements in the periodic table that are named after the country, where they were first
made or obtained. For example, the Latin name for copper was coined by the Romans because their
chief source of copper was from the Island of Cyprus. However, there is one country in the world
which was named after an element (the Latin name). A long time ago, it was believed that this country
had mountains full of a valuable element, however all expeditions to find these mountains failed. But
the name stuck on. The element in question is used for many applications today, and many of its
compounds are used as catalysts. The ions of this metal have very good anti-microbial property and
finds application in water purification. The element is
(a) Sodium (b) Gold (c) Silver (d) Francium
Ans. (c) Silver
Sol. Argentina is named after silver "Argentuns"

3
12. All of these species have the same number of valence electrons as nitrate ion, except
(a) Carbonate ion (b) Bicarbonate ion
(c) NF 3 (d) SO3
Ans. (c) NF 3
Sol. No. of valence e–s in nitrate ion is
NO3– = 24.
(a) CO 3 2– : Valence e– = 24
(b) HCO 3 – : Valence e– = 24
(c) NF 3 : Valence e– = 26
(d) SO3 : Valence e– = 24
13. The angle between the hour arm and the minute arm of a clock at 2:10 a.m. is
(a) Zero (b) 4° (c) 5° (d) 6°
Ans. (c)

11M - 60H
Sol.. Angle between hour hand and min hand is given by
2
Where M = min. and H = Hour

11 ´ 10 - 2 ´ 60
So, angle between hour hand and Min hand of 2 : 10 AM is = 5º
2
Option is (c) correct
14. A craft teacher reshapes the wax from a cylinder of candle with section diameter 6 cm and the height
6cm into a sphere. The radius of this sphere will be

(a) r = 6 3 / 2 cm (b) r = 6 cm

(c) r = 3 3 3 / 2 cm (d) r = 6 3 2 cm
Ans. (c)
Sol. Volume of wax in cylindrial shape = volume of wax is spherical shape
= pr2h = 4/3pR3
22 4
= × (3)2 × 6 = pR3
7 3
32 ´ 6 ´ 3
Þ R3 =
4
3
Þ R = 33 cm
2
Option (c) is correct
15. Plants absorb nitrates from the soil, which are most essential to produce
(a) Proteins (b) Carbohydrates
(c) Fats (d) Cell wall
Ans. (a)
Sol. Plant absorb nitrates from the soil which is essential to produce protein.

4
16. The dry mass (mass excluding water) of a seed in the process of germination
(a) increases over time until the first leaves appear
(b) decreases over time until the first leaves appear
(c) stays constant until the first leaves appear
(d) first increases and then decreases until the first leaves appear.
Ans. (b)
Sol. During the time of germination the dry mass of seed will decrease, because it will do respiration.
17. A point object O is kept at origin. When a concave mirror M1 placed at x = 6 cm, image is formed at
infinity. When M1 is replaced by another concave mirror M2 at same position, image is formed at x =
30 cm, then ratio of the focal length of M1 to that of M2 is

3 4 1
(a) (b) (c) 5 (d)
4 3 5
Ans. (a)
Sol. Since image is formed at ¥
fM1 = –6
For M2,
1 1 1
- =
(30 - 6) 6 fM2
1 1 1
or 24 - 6 = f
M2

1- 4 1
=
24 fM2

Þ fM2 = –8 cm
fM1 6 3
\ f =8=4
M2

18. The number 38 (310 + 65) + 23(212 + 67) is


(a) A perfect square and a perfect cube
(b) Neither a perfect squre nor a perfect cube
(c) A perfect cube but not a perfect square
(d) A perfect square but not a perfect cube
Ans. (c)
Sol. 38(310 + 65) + 23(212 + 67)
318 + 38.35.25 + 215 + 210.37
318 + 313.25 + 37210 + 215
318 + 37.25 (36 + 25) + 215
(36)3 + 3(36)(25) (36 + 25) + (25)3
(36 + 25)3 which is perfect cube.
It is a perfect cube but not a perfect square
So, option (c) is correct.

5
19. Melting point of a substance is 10°C. What does this mean?
(a) The substance is a liquid at 10°C.
(b) The substance is a solid at 10°C.
(c) There is an equilibrium between solid phase and liquid phase at 10°C
(d) The substance is 50% solid and 50% liquid at 10°C.
Ans. (c)
Sol. Melting opint is defiend as the temperature at which solid and liquid phase of a substance are in state
of dynamic equilbrium at a fixed pressure.
20. The following substances have approximately same molecular mass. Which is likely to have the
highest boiling point?
(a) n-butane (b) Isobutane (c) n-butanol (d) Isobutanol
Ans. (c) n- butanol.
Sol. B.P. of n – Butanol = 117.7°C
B.P. of iso – Butanol = 108°C
B.P. of n – Butane = –1°C
B.P. of iso – Butane = –11.7°C
21. U-tube contains some amount of mercury. Immiscible Liquid X is poured in left Immiscible liquid Y
is poured in the right arm. length of liquid X is 8 cm. length Y is 10 cm and upper levels of X and Y are
equal. If density of Y is 3.36 g.cm–3 and l3.6 g.cm–3 then density of X is
arm of
X Y liquid of
Hg is

(a) 0.8 g-cm–1 (b) 1.2 g-cm–1 (c) 1.4 g-cm–1 (d) 16 g-cm–1
Ans. (a)
Sol. PA = PB
P0 + rxg(8) + rHgg(2) = rY × G × 10 + P0 8 cm X
2 cm Y 10 cm
8rX = 10rY – 2rHg = 10 × 3.36 – 2 × 13.6 = 33.6 – 27.2
A B
8rX = 6.4
6.4
rX = = 0.8 g cm–3
8
22. Let the number of rectangles formed by 6 horizontal and 4 vertical lines be n and those formed by 5
vertical and 5 horizontal lines be m then we have
(a) n = m (b) n ³ m + l (c) m ³ n (d) m > n + 5
Ans. (d)
Sol. Number of rectangles formed by 6 horizontal and 4 vertical lines is 6C × 4C = 90
2 2
\ n = 90
Also number of rectangles formed by 5 vertical and 5 horizontal lines is 5C × 5C = 100
2 2
Þ m = 100
\m>n+5
so, option (d) is correct.
23. In a human cell undergoing Meiosis, what are the total number of cellular DNA molecules present
during Prophase-l
(a) 23 (b) 46 (c) 69 (d) 92
Ans. (d)
Sol. During the interphase the process of DNA replication will occur. That's why the DNA content in
prophase-I will be doubled.
6
24. During gaseous exchange in the alveoli, what happens to nitrogen?
(a) There is no net nitrogen exchange, as nitrogen is filtered out by the alveoli.
(b) The nitrogen is absorbed by the alveolus to form amino acids.
(c) The nitrogen is filtered out by the alveolus, as the nitrogen molecule is too large to cross the gaps
in the capillaries
(d) There is no net nitrogen exchange, as the blood is saturated with nitrogen
Ans. (d)
Sol. There is no net nitrogen exchange as, the blood is saturated with nitrogen.
25. The effective resistance between A and D in the circuit shown in the adjacent figure is
5W 10W 5W
A B

10W 10W

C D
5W 10W 5W
(a) 5W (b) 10 W (c) 15 W (d) 20 W
Ans. (d)
Sol. From the fig.
5W P 10W
A A
5W
10W 10W 10W
Þ
D
D 5W
10W Q 5W
Equivalent resistance
Between P and Q is 10W
Req. = 5 + 10 + 5
= 20 W
26. If ABCD is a rhombus and ÐABC = 60° then
(a) The points A,B,C,D are concyclic
(b) The quadrilateral has exactly half the area of the square with same sides as ABCD
3
(c) The quadrilateral has area AB2
2
(d) The diagonals of the quadrilateral ABCD are equal and bisect each other at right angle
Ans. (c)
Sol. Here
AM = sin 60º × a A D
a
3a 60º
=
2 B M C
a
3a
\ Area of rhombus = Base × height = a ×
2

3a 2
=
2

3
= (AB)2
2
so, option (c) is correct.

7
27. Identify the overall change in the following set of reactions:
(1) Carbon dioxide ® carbonic acid (H2 CO3 ) (2) Ethanol (alcohol) ® Ethanal (aldehyde)
(3) Ethanal (aldehyde) ® Ethanol (alcohol) (4) Sulphuric acid ® Sulphur trloxlde (SO3 )
Choose the correct option which best describes these conversions
(a) oxidation, oxidation, reduction, reduction
(b) hydration, oxidation, reduction, dehydration
(c) reduction, dehydration, hydration, oxidation
(d) reduction, reduction, oxidation, oxidation
Ans. (b)
Sol. (1) CO2 + H2O ® H2CO3 : Hydration
(2) C2H5OH ® CH3CHO : Oxidation
(3) CH3CHO ® CH3CH2 OH : Reduction
(4) H2SO4 ® SO3 + H2O : Dehydration

28. An element with atomic number 44, is below which element in the periodic table?
(a) Calcium (b) Iron (c) Argon (d) Magnesium
Ans. (b) Fe
Sol. At. No. 44 "Ruthenium" is below Fe in the peridic table
29. Three bulbs B1 , B2 and B3 having rated powers 100 W, 60 W and 60 W at 250 V are connected in a
circuit as shown in the adjacent figure. If W1, W2 and W3 are the output powers of the bulbs B1 , B2 and
B3 respectively, then

B1 B2

B3

250V

(a) W1 > W2 = W3 (b) W1 > W2 > W3 (c) W1 < W2 = W3 (d) W1 < W2 < W3
Ans. (d)
Sol. Since power rating is provided.

B1 B2

i1 B3

i2

250V

R B1 < R B2 = R B3
From the given circuit current through B3 will be more than that through B1 and B2. (i1 < i2) So, after
comparing power from equation i2R. We get W1 < W2 < W3 , which is option (d).
30. If a, b > 0 then
(a) a + b £ ab (b) a + b > ab
(c) a + b ³ 2ab (d) None of the above inequalities will hold

8
Ans. (d)
Sol. If a, b > 0
Then AM ³ GM
a+b
Þ ³ ab
2

Þ a + b ³ 2 ab Þ a + b > ab
So option (b) is correct.
31. Which of the following is true about ATP
(a) It is a derivative of one of the nitrogenous bases that form DNA
(b) It splits into ADP and phosphate and the energy produced is used by muscle cells to contract
(c) It is produced in both aerobic and anaerobic conditions.
(d) All of the above
Ans. (d)
Sol. ATP is derivative of Adenosine, ATP is also used in muscles contraction, ATP is also produced in both
aerobic & anaerobic condition. That's why the answer will be all of the above.
32. Which of the following is true regarding communication in neurons
(a) Free electrons are moved along the plasma membrane of the axon and control the expression of
neurotransmitters
(b) A chemical signal travels along the axon and is converted into an electric impulse at the synapse
(c) An electric impulse travels along the length of the axon. The electric impulse is converted to a
chemical signal at the synapse.
(d) An electrical signal is converted to a chemical signal by the Myelin sheath before it reaches the
synnapse
Ans. (c)
Sol. An electric impulse travels along the length of the axon. The electric impulse is converted to a chemical
signal with the help of neurotransmitter “Acetyl choline” at synapse.

33. Following diagram shows refraction of parallelo beam of light through a spherical surface. Identify
the correct ray diagram

Rarer Denser
Denser Rarer

(a) (b)

Rarer
Denser

(c) (d)

Ans. (b)
Sol. As rays bend towards normal in denser medium so, option (b) is correct.

9
34. Tenth term in the sequence 12, 18, 20, 28, ... is
(a) 336 (b) 63 (c) 216 (d) 68
Ans. (b)
Sol. 12,18,20,28......
22 × 3, 32 × 2, 22 × 5, 22 × 7,.............
These are the number with 6 factors so next possible terms with 6 factors are
25 × 1, 22 × 11, 32 × 5, 52 × 2, 22 × 13, 32 × 7, .............
\ 10th term is 63.
So option (b) is correct.
35. An electron pair donor is a Lewis base and an electron pair acceptor is a Lewis acid. Which among the
following statements, is correct?
(a) NH3 is a Lewis acid, because nitrogen has only 6 electrons around it
(b) BF3 is a Lewis base, because fluorine has 8 electrons around it.
(c) NF3 is a Lewis base, because nitrogen has a lone electron.
(d) BCI3 is a Lewis acid because it has only 6 electrons around it.
Ans. (d)
Sol. (d) most appropriate
(c) Option has lone e– not lone pair of e–s.
36. Greenhouse gases absorb (and trap) outgoing infared radiation (heat) from Earth and contribute to
global warming. A molecule that acts as a greenhouse gas, generally has a permanent dipole moment
and sometimes for other reasons. Going only by the condition of permanent dipole moment, in the list
of gases given below, how many can be potential greenhouse gases?
Water, Sulphur dioxide, Boron trifluoride, carbon monoxide, carbon dioxide, nitrogen, oxygen, methane,
hydrogen sulphide, ammonia.
(a) Five (b) Six (c) Seven (d) Four
Ans. (a) five
Sol. Water, SO2, CO, H2S and NH3 will have permanent dipole moment
37. In the diagram M1 and M2 are two plane mirrors at right angles to each other. O is a luminous point
object. Consider two images formed due to first reflection at M1 and M2. The area of the triangle
formed by the object and two images is

2 cm M1
O

2 cm

M2

(a) 4 cm2 (b) 2 cm2 (c) 8 cm2 (d) 16 cm2


Ans. (c)
O I1
1 2 cm 2 cm
Sol. Area of DOI1I2 = × I1 × OI2
2
2 cm

1
= ×4×4 2 cm
2
= 8 cm2 I2
10
38. The probability of a point within an equilateral triangle with side 1-unit lying outside its in-circle
(inscribed circle) is
1 p p 2p
(a) 1 – (b) 1 – (c) l – (d) 1 –
2( 3) 3 3 2 3 3 3
Ans. (b)
A
D
Sol. Radius of in-circle =
S

3 2
(1) 1
= 4 =
3 2 3
2 B C

3 2
(1) - p(r 2 ) p
\ Probability of a point lying outside the circle = 4 = 1–
3 2 3 3
(1)
4
So option (b) is correct.
39. Penicillin cannot be used to treat influenza because:
(a) It only helps to bring the temperature down, and does not reduce the infection
(b) The penicillin is broken down by the organism
(c) Viruses do not have cell walls
(d) Reproduction of protozoans is not affected by penicillin
Ans. (c)
Sol. Antibiotics are not used in viral disease, because virus do not have cell wall.
40. Thin cuboidal strips are made by slicing a potato. They are all made to be exactly 8 cm long and 2 mm
wide. Each strip is placed in sugar solutions of different concentration. After soaking it for 5 hours,
their lengths are measured again. The following graph shows the results of the experiment. What
concentration of sugar solution is isotonic with the contents of the cells of the potato.

8.5
8.4
8.3
8.2
8.1
8
7.9
7.8
7.7
7.6
0 0.1 0.2 0.3 0.4 0.5 0.6 0.7 0.8

(a) 0.2 (b) 0.4 (c) 0.6 (d) 0.1


Ans. (b)
Sol. In the mention graph, 8 cm length of the slice will be on 0.4 concentrate solution [Isotonic] which will
be similar to the initial slice length 8 cm.

11
41. A fisherman of height h is standing on the bank of a lake. A fish in the water perceives his height as h'.
Then
(a) h' > h (b) h' < h
(c) h' = h (d) h' > h or h' < h depending on position of fish
Ans. (a)

Sol.

h'
h

From Fig. itself h' = mh


42. A triangle has perimeter 316 and its sides are of integer length. The maximum possible area for such
a triangle is achieved for
(a) Single triangle (b) Two triangles
(c) Three triangles (d) More than three triangle
Ans. (a)
Sol. a + b + c = 316 and a, b, c Î I
Now, area is maximum only if a = b = c
But here sides are integers,

316
so it is not possible as 3a = 316 Þ a = = 105.33
3
\ Area is Max. only when a = b = 105 and c = 106.
So option (a) is correct.
43. Hennig Brand, one of the many alchemists was in pursuit of the philosopher's stone. Brand's method
is believed to have consisted of evaporating large quantities of urine to leave a black residue that was
then left for a few months. The residue was then heated with sand, driving off a variety of gases and
oils. The final substance to be driven off, was condensed as a white solid, which he called as "cold
fire" as it was luminousand glowed in the dark and also caught fire on slight warming and producing
a large quantity of light. It has also been called as the "Bearer of light". Which element is "cold fire"?
(a) Lithium (b) Tungsten (c) Phosphorous (d) Cesium
Ans. (c)
Sol. Phosphorous is called cold fire.
44. When solid KOH is mixed with solid NH4Cl, a gas is produced. Which gas is it?
(a) Chlorine (b) hydrogen (c) hydrogen chloride (d) ammonia
Ans. (d) ammonia
Sol. KOH + NH4Cl ® KCl + H2O + NH3 ­
12
45. Object A is completely immersed in water. True weight of object A is WA. Weight of water with beaker
is WB. Let B be the buoyant force. W1 and W2 are scale readings of spring balance and weighing scale
respectively.

Spring balance

Weighing scale
....... gm

(a) W1 = WA (b) W1 = WA + B (c) W2 = WB (d) W2 = WB + B


Ans. (d)
Sol. For spring balance
kx = WA – B kx B
Þ WA – B ....... (1) N
wB
For weighing machine Spring A
balance Weighline
N = B + WB
B Scale
Þ W2 = WB + B .......(2)
Hence ‘d’ is correct mg = wA

46. Number of numbes less than 40 having exaclty four divisors is


(a) 15 (b) 12 (c) 11 (d) 14
Ans. (d)
Sol. No having 4 divisor should be is the form of a11 × a21 or a13
\ Possible Numbers are

6=2×3 15 = 3 × 5
10 = 2 × 5 21 = 3 × 7
14 = 2 × 7 33 = 3 × 11
22 = 2 × 11 39 = 3 × 13
26 = 2 × 13 35 = 5 × 7
34 = 2 × 17 8 = 23
38 = 2 × 19 27 = 33

So, option (d) is correct.


47. Antibodies play an important role in defending the body agains infections by which of the following
meachanism:
(a) They engulf the bacteria and make them harmelss
(b) They bind to the surface of pathogens, so taht they can be easily indentified and removed by other
cells of the immune system
(c) They enter the pathogen and prevent cell division
(d) They are highlty reactive and chemically react with the DNA of the pathogen
Ans. (b)
Sol. Antibody, Binds to the surface of pathogen, so that they can be easily identified & removed by other
cells of Immune system.

13
48. The figure shows a food we, where A, B, C, D etc. are different species. And the direction of the
arrows symbolizes the direction of flow of natrients
An increase in the population of which species is likely to decrease the population of species.

A B

C D

E F
G

(a) Species D (b) Sepecies F (c) Species G (d) Species E


Ans. (a)
Sol. As mention in the figure, “C” is eaten by “A”, “E” is eaten by both “C” & “D”, that's why if we
increase the number of “D”, the number of “E” will be decrease similarly “C” & “A” also decrease.
49. Velocity of a particle moving along a straight line varies with time as shown in the figure. Net forces
acting on the aprticle are F1, F2, F3, F4 and F5 in the intervals OA, AB, BC, CD and DE respectively.
Indentify the correct statement
V

A
D
t
B C E

(a) F1 increases with time (b) F5 is initially positive and then becomes negative
(c) F1 and F2 are in opposite directions (d) F3 is negative
Ans. (c)
e
+v

v
Sol. c
st =

D
on

A
=c
F1

F2 = ve F5 = ve
F
O
F4 = +ve
F3 = 0
B C

Q Slope of the velocity time gives


us acceleration and F = ma
option ‘c’ is correct.
50. If set X consists of three elements then the number of elements in the power set of power set of X is
(a) 3 3 (b) 23 (c) 38 (d) 28
Ans. (d)
Sol. Let X = {a, b c}
Then No. of elements in the power set of A = 23 = 8
\ No. of element in the power set of power set of A = 28
So, option (d) is correct.

14
51. The heat of neutralisation of CH3COOH, HCOOH, HCN and H2S are – 55. 2, – 56.07 – 2.8 and – 3.34
kJ per equaivalent respectively. The increasing order of strength of these acids is
(a) HCOOH < CH3COOH < H2S < HCN (b) H2S < HCN < HCOOH < CH3COOH
(c) HCN < H2S < CH3COOH < HCOOH (d) CH3COOH < HCOOH < HCN < H2S
Ans. (c) HCN < H2S < CH3COOH < HCOOH
Sol. Higher the heat of neutralisation stronger will be the acid
52. To prevent the formation of oxides, peroxides, and superoxides, alkali metals are sometimes stored in
an unreactive atomosphere. Which of the following gases should not be used for lithium :
(a) Ne (b) Ar (c) N2 (d) Kr
Ans. (c) N 2
Sol. As Li can react with N2 forming explosive Li3N.
53. A wooden block (W) is suspended by using a coard from aheavy steel ball (B). The entire system is
dropped from a height. Neglecting air resistance, the tension in the cord is
(a) Zero (b) The difference in the masses of B and W
(c) The differences in the weights of B and W (d) The weights of B

Ans. (a)
B

Sol. For wooden block


T g

W – T = mg

Þ T=0 W

54. In a n- sided regular polygon, the radius of the circum-circle is equal in length to the shortest diagonal.
the number of values of n < 60 for which this can happen is
(a) 0 (b) 1 (c) 10 (d) 2
Ans. (b)
Sol. DOCE is an equilateral triangle.
So ÐCOE = 60°
DOCP @ DOEP A
So ÐDOE = 30° B
Angle made at centre by the side of regular polygon = 30° G
R O
360° C
So, = 30°
n R R F
P
D
360° E
n= = 12
30
So, option (b) is correct.
55. Which of the following does NOT contain living cells?
(a) Bone tissue (b) Xylem sieve tubes (c) Phloem (d) epidermis
Ans. (b)
Sol. Xylem sieve tubes do not contain living cells.
56. If DNA was made of 6 nucleotides instead of 4, what are the total number of triplet codons possible?
(a) 24 (b) 18 (c) 64 (d) 216
Ans. (d)
Sol. If we have 4 nitrogen base then we have 64 codons (triplet) 43 Þ 64, As in question there are 6
nucleotide, then 63 = 216.

15
57. A circus performer of weight W is standing on a wire as shown in the figure. The tension in the wire
is

w w w w
(a) Approximately (b) Approximately (c) Much more than (d) Much less than
4 2 2 2
Ans. (c)
Sol. From equilibrium condition
W = 2T sinq
q q
W T T
or T =
2sin q W

W
Since q is small, T will be much more than .
2
58. Number of integers n such that the number 1 + n is a divisor of the number 1 + n2 is :
(a) 0 (b) 1 (c) 4 (d) 2
Ans. (c)
Sol. n2 + 1 = n2 – 1 + 2 = (n – 1)(n + 1) + 2
Now, n2 + 1 is divisible by n + 1 only when 2 is divisible by n + 1
So, n = –3, –2, 0, 1
So, option (c) is correct.
59. When 1 gram of a mixture of aluminium and zinc was treated with HCl, a gas was liberated. At the end
of the reaction the volume of the liberated gas was found to be 524 cm3 , under STP conditions. The
individual weight of aluminium and zinc in the mixture respectively are :
(a) 0.2 g and 0.8 g (b) 0.8 g and 0.2 g (c) 0.5 g and 0.5 g (d) 0.322 g and 0.678 g
Ans. (a)
Sol. Al = xg Zn = 1 – x g
3
Al + 3HCl ® AlCl3 + H
2 2

x 3 x
´ moles
27 2 27
Zn + 2HCl ® ZnCl2 + H2
1- x 1- x
moles.
65.3 65.3

x (1 - x) 524
Total H2produced = + moles = (give)
e)
18 65.3 22400
Þ Solving this x = 0.21 gm and Zn = 0.79 gm
60. Choose that element which is most different from the other three.
(a) Carbon (b) Nitrogen (c) Silicon (d) Phosphorous
Ans. (a)
Sol. Carbon. due to is special character forming organic chemistry.

16
61. In the following diagrams O is point object and I is its image formed by a concave mirror. Identify the
diagram. In which position of image I is nearly correct.

I I
C F C F C F C F

(a) (b) (c) (d)


O O O O I
I

Ans. (a)
Sol. From ray diagram, it is clear that option ‘a’ is correct.

C F q
q
O

1 1
62. If for x, y > 0 we have + = 2 then the minimum value of xy is
x y

(a) 2 (b) 1 (c) 4 (d) 2


Ans. (b)

1 1
Sol. If + =2
x y

Now, AM ³ GM

1 1
+
x y 1
Þ ³
2 xy

1 2
Þ £
xy 2

1
Þ £1
xy

Þ xy ³ 1
Hence least value of xy = 1
So, option (b) is correct.
63. Which of these is a mollusc?
(a) Lobster (b) Scorpion (c) Crab (d) Octopus
Ans. (d)
Sol. “Octopus” Belongs to mollusc

17
64. What is the mechanism used by the kindly to remove waste products from the body?
(a) Nephrons convert nitrogenous waste to uric acid and pass it out as urine
(b) Nephrons actively transport uric acid and other nitrogenous waste A into the proximal and distal
convoluted tubules, from where it is collected to form urine.
(c) The blood is filtered to retain cells and large plasma proteins within the blood. The remaining
filtrate passes through the proximal and distal convoluted tubules, where needed substances are
reabsorbed by active transport
(d) Nephrons filter out the nitrogenous waste which is passed through the proximal and distal convoluted
tubules and collected by the collecting duct as urine.
Ans. (c)
Sol. In kidney blood cells & plasma will not enter in PCT & the remaining filterate is passed through the
Nephron, & all useful materials are re-absorbed actively.
65. Two bodies A and B are charged with equal magnitude of charge but A with positive charge and B
with negative. If MA and MB are masses before charging and MIA and MIB are the masses after charging
the (m0 is some constant mass)
(a) MIA = MA + m0 and MIB = MB – m0 (b) MIA = MA – m0 and MIB = MB + m0

m0
(c) MIA = MIB (d) MIA = MA – and MIB = MB + m0
2
Ans. (b)
Sol. If a particle is positively charged then electron leaves the object. If particle is negatively charged,
extra electron comes into it.
Q M IA = M A – m 0 , M IB = M B + m 0

66. The number of natural numbers n £ 30 for for which n + n + n + ..... is a natural number is
(a) 30 (b) Zero (c) 6 (d) 5
Ans. (d)

Sol. Let K = n + n + n + .....

Þ K= n+K
Þ K =n+K
2

Þ K2 – n – K = 0

1 + 1 + 4n
Þ K= is a natural no only
2
When 4n + 1 is a perfect square
Q n = 2, 6, 12, 20, 30
So, option (d) is correct.
67. Elements A, B and C have atomic numbers X, X + 1 and X + 2, respectively. 'C' is an alkali metal 'A'
reacts with another element 'Y' to form the compound 'AY'. 'A' and 'Y' belong to the same group. 'AY'
possesses an
(a) ionic bond (b) covalent bond (c) metallic bonding (d) coordinate bond
Ans. (b) Covalent bond
Sol. AY is an interhalogen compound Ex ICl

18
68. Air has three major components : nitrogen, oxygen and argon. Given that one mole of air at sea level
is made up of 78% nitrogen, 21% oxygen, and 1% argon, by volume. What is the density of air?
Assume that gases behave in an ideal manner. (Atomic mass of argon is 40 amu)
(a) 14.62 g/L (b) 1.3 g/L (c) 29 g/L (d) 0.65 g/L
Ans. (b) 1.3 gm/L
78 ´ 28 + 21 ´ 32 + 1´ 40
Sol. Molar mass of air = = 28.96 gm
100
mass
density =
volume
1 moles of air will occupy 22.4 litre.
28.96 gm
\ d= = 1.29 gm/L.
22.4 lit
69. A conductor of length L has a varying cross section with area 2A at P and A at Q as shown in the
figure. If it carries a steady current I, then
P
Q

L
(a) Net charge per unit volume near P is more than net charge per unit volume near Q.
(b) Net charge per unit volume near Q is less than net charge per unit volume.
(c) Current per unit area near P is more than current per unit area near Q.
(d) Current per unit area near P is less than current per unit area near Q.
70. The number of natural numbers n £ 30 for which n + n + n + ...... is a prime number is
(a) Three (b) Zero (c) Nine (d) Two
Ans. (a)

1 + 1 + 4n
Sol. Let K = n + n + n + .... Þ K = 2
Here, K is prime only when n = 2,6,20
So, option (a) is correct.
71. Rhodoferax fermentans is a species of photosynthetic bacteria. From your knowledge about bacteria
in general, identify the components that CANNOT be present in this organism.
(a) Chloroplasts (b) ATP
(b) Ribosomes (d) Cell wall
Ans. (a)
Sol. Bacteria do not have membrane bound cell - organelles, that's why chloroplast will be absent in
bacteria.
72. If atmospheric humidity decreases, transpiration rate
(a) Decreases because the concentration gradient between the mesophyll and the atmosphere decreases.
(b) Stays the same because active transport does not depend on humidity
(c) Increases because of the higher concentration gradient between the air spaces of the mesophyll
and the atmosphere
(d) Decreases because the concentration of water vapour decreases
Ans. (c)
Sol. If humudity decreases the transpiration rate will increase, because there will be more water in mesophyll
cell as compared to atmosphere.
19
73. Vessels A and B are made of conducting material. Both contain water. Vessel A floats in B. Vessel B is
now heated at a uniform rate, then
(a) Water in A boils first.
(b) Water in A boils some time after water in B starts boiling.
(c) Water in both A and B start boiling simultaneously.
(d) Water in A does not boil.

Ans. (d)

Sol. As water in vessel A gets heat via conduction through the vessel, so when temp. reaches 100°C on
both inside and outside of vessel A, conduction stops and the water inside, does not get latent heat for
boiling whereas the vessel B gets continuous heat from the burner for boiling. So water in A does not
boil.
74. The number of squares formed by 5 vertical and 4 horizontal lines (all are equispaced) is
(a) 60 (b) 20 (c) 40 (d) 46
Ans. (d)
Sol. No. of square formed by 5 vertical and 4 horizontal lines
= 4 × 3 + 3 × 2 + 2 × 1 = 20
So, option (d) is correct.
75. If 0.50 mole of a monovalent metal (M+1 ) halide is mixed with 0.2 mole of a divalent metal (L+2 )
phosphate, the maximum number of moles of M3PO4 that can be formed is
(a) 0.25 (b) 0.30 (c) 0.16 (d) 0.20
Ans. (c) 0.166 mole of M3PO4
Sol. 6MX + L3(PO4)2 ® 2M3PO 4 + 3LX2
MX is acting as Limiting meganet.
76. Every major city in India has a pollution control board to monitor air and water pollution. The following
data is from three different localities in Bangalore city from the year 2015.

Locality Annual average of SO2 in the air


(volume/volume)
P 16.3 mL/m 3
Y 16.3 ppb (m 3/m 3)
Z 16.3 ppm (m3/m3)
ppb stands for parts per billion and ppm stands for parts per million. These are different units to
express concentration. They are very similar to percentage (which is actually parts per hundred).
Based on the above data, which place will you choose to live in?
(a) All localities are equally polluted, so I have no preference.
(b) P is the more polluted than Y and Z, hence I will live in either Y or Z.
(c) Locality Y is least polluted, hence I will live in Y.
(d) Z and Y are more polluted than P, hence I will live in P.

20
Ans. (c) Locality Y is least polluted, hence I will live in Y.
ml 16.3ml
Sol. P = 16.3 3 = =16.3 ppm ml/ml
m 106 ml
16.3 m3
Y = 16.3 ppb =
109 m3
16.3 m3
Z = 16.3 ppm = = 16.3 ppm
106 m3
77. A body thrown vertically up reaches a maximum height and returns back. Its acceleration is
(a) Downward during both ascent and descent.
(b) Downward at all positions except at the highest point, where it is zero.
(c) Upward during both ascent and descent.
(d) Downward during ascent and upward during descent.
Ans. (a)
Sol. In both cases acceleration will be in downward direction and equal to ‘g’ if we neglect air friction.
78. The number of integers a, b, c for which a2 + b2 – 8c = 3 is
(a) 2 (b) Infinite (c) 0 (d) 4
Ans. (c)
Sol. If a2 + b2 – 8c = 3
so a2 + b2 = 8c + 3
Case I : If both a = even, b = even
a = 2m, b = 2n
4m2 + 4n2 = 8c + 3
LHS = Even , But RHS = odd
Hence no solution
Like wise for both a = odd , b = odd., There is no solution.
Case II :If any one number is odd,
Let a = 2m, b = 2n + 1
so, (2m)2 + (2n + 1)2 = 8c + 3
4 m2 + 4n2 + 4n + 1 = 8c + 3
4(m2 + n2 + n) = 8c + 2
LHS is divisible by 4
But RHS is not divisible by 4.
Hence no solution.
So, option (c) is correct.
79. Which of the following is NOT produced by the endoplasmic reticulum?
(a) Lipids (b) Proteins (c) Monosaccharides (d) Hormones
Ans. (c)
Sol. Monosaccharides, do not produced by endoplasmic reticulum.
80. Vaccines prevent infections by pathogens by:
(a) Presenting the body's immune system with antigens in a controlled manner, so that it is prepared to
counter the pathogen producing it when it attempts to infect the body
(b) Affecting the reproductive cycle of the invading pathogen
(c) Binding to antigens on the surface of the pathogen and inactivating it
(d) Affecting the metabolic pathways of the pathogen
Ans. (a)
Sol. Vaccines stimulates formation of antibody in our body to counters with the upcoming pathogen.

21
NSEJS-2017 (IJSO STAGE-I)
Date of Examination : 19th November, 2017
PAPER CODE - JS531

SOLUTIONS
1. Rajiv, Nikhil, Shubha and Nilima wanted to establish a relationship between loss in weight of a solid
with weight of water displaced by immersing it in tap water and sea water. After performing their
experiment, they noted their observations for the same solid as follows :
Rajiv : Loss of weight of solid is more in tap water.
Nikhil : Loss of weight of solid is more in sea water.
Shubha : Loss of weight of solid is equal in the tap water and the sea water.
Nilima : Loss of weight of solid may be more in tap water or sea water, depending upon how deeply
it is immersed, identify the correct observation.
(a) Nikhil (b) Nilima
(c) Shubha (d) Rajiv
Ans. (a)
Sol. dsea water > dtap water
So, weight of water displaced in sea water > in tap water.
2. The ratio of atoms present in 4 g of magnesium and 4 g of sulphur is (Mg = 24 ; S = 32)
(a) 1 : 1 (b) 2 : 1
(c) 3 : 2 (d) 3 : 4
Ans. (d)
4
´1
24 4
Sol. Ratio = 4 =
3
32
3. If Z = 10 the valency of the element is..............
(a) zero (b) one
(c) two (d) three
Ans. (a)
Sol. Z = 10 is for neon (noble gas)
Hence, valency is zero.
4. The average atomic mass of an element X is 80 u. The present of isotopes 79
X35 and 82
X35 in the
sample is
(a) 90.99 and 9.01 (b) 80.8 and 19.2
(c) 66.67 and 33.34 (d) 50 and 50
Ans. (c)
(79 ´ x) + 82 ´ (100 - x)
Sol. 80 =
100
x = 66.67%
% 79X35 = 66.67% & % 82X35 = 33.34%

1
5. An aqueous solution used to preserve biological specimen is
(a) Methane (b) Methanol
(c) Methanal (d) Methanoic acid
Ans. (c)
Sol. Biological specimen is stored in HCHO.
6. The molecular formulae of some organic compounds are given below, which of these compounds
contains a Ketone group ?
(a) C3 H6 O 2 (b) C3 H6 O
(c) C3 H4 O (d) C3 H8 O
Ans. (b)
Sol. Ketone has D.U. = 1
7. 'Duralumin' is an alloy of aluminium with
(a) iron, manganese and magnesium (b) copper, manganese and magnesium
(c) copper, chromium and magnesium (d) iron, nickel and magnesium
Ans. (b)
Sol. Duralumin contains : Al - 95%, Cu - 4%, Mg - 0.5%, Mn - 0.5%
8. Tooth decay starts when the pH around tooth is around
(a) 7.5 (b) 7
(c) 6.5 (d) 5.5
Ans. (d)
Sol. Tooth decay occurs in acidic medium, pH = 5.5.
9. What will happen if a copper piece is dipped in aqueous solution of silver nitrate for quite some time?
(i) Solution will remain colourless
(ii) Solution will turn blue
(iii) Silver will deposit on the copper piece
(iv) Bubbles of brown gas will be formed around copper piece
(a) i and iv (b) ii and iv
(c) ii and iii (d) iii and iv
Ans. (c)
Sol. Copper being more reactive than silver will displace it, so silver will bet deposited & solution turns
blue because of formation of Cu2+.
Cu + AgNO3 ® Cu2+ + Ag¯ + NO3–
Blue
10. Neeta mixed 10 mL of 0.1 M HCl solution with 15 mL of 0.067 M NaOH solution. She checked the
pH of the resulting solution using pH paper. The colour obtained was
Red Orange Yellow Green Pale blue Dark blue Violet
Strong acid ¬¾¾ Weak acid Neutral Weak alkali ¬¾¾ Strong alkali
(a) Green (b) Yellow
(c) Pale blue (d) Violet
Ans. (c)
Sol. HCl + NaOH ® NaCl + H2O
10 × 0.1 15 × 0.067
1mmol 1.005 mmol

2
0.005
Solution is basic, [OH–] = = 2 × 10–4
25
pOH = 3.7
pH = 10.3 Pale blue – Dark blue
Appropriate answer (c)
11. (I) Zn + CuSO4(aq) ® Reaction occurs
(II) Zn + Al2(SO4)3 (aq) ® Reaction does not occur
(III) Zn + AgNO3 (aq) ® Reaction does not occur
(IV) Zn + PbNO3(aq) ® Reaction occurs
Which of the above statements is not correct ?
(a) I (b) II (c) III (d) IV
Ans. (c)
Sol. (I) Zn + CuSO4(aq) ® ZnSO4 + Cu – True
(II) Zn + Al2(SO4)3 (aq) ® No reaction – True
(III) Zn + AgNO3 ® Zn(NO3)2 + Ag – False
(IV) Zn + Pb(NO3)2 ® Zn (NO3) + Pb – True
12. An open vessel contains air at 27°C. The vessel is heated till two-fifth of the air in it has been expelled.
Assuming the volume of the vessel remains constant, find the temperature to which the vessel has to
be heated ?
(a) 750 K (b) 700 K (c) 550 K (d) 500 K
Ans. (d)
Sol. T = 300 K;
n1T1 = n2T2
3
n × 300 = × n × T2
5
Þ T2 = 500 K
13. A teacher wanted to give acid base titration to her students. For that she prepared (i) HCl solution by
dissolving 73 g of hydrochloric acid in one litre of water and (ii) sodium hydroxide solution by
dissolving 0.46 g of sodium metal in one litre of water. Find the volume of the hydrochloric acid
solution required for complete neutralisation of sodium hydroxide solution. (Cl = 35.5 ; Na = 23.0 ; O
= 16.0)
(a) 20 mL (b) 10 mL (c) 46 mL (d) 5 mL
Ans. (b)

73
Sol. [HCl] = = 2M
36.5
1
Na + H2O ¾® NaOH + 1H2
0.46
23
[NaOH] = 0.02 M
0.02 × 1 = 2 × v
v = 0.01 L = 10 mL

3
14. What would be the atomic number of the next halogen element, if discovered in future ?
(a) 103 (b) 115 (c) 117 (d) 121
Ans. (c)
Sol. After Astatine next element will have atomic number = 85 + 32 = 117.
15. A white solid known to be a compound of sodium, given rise to water vapour and a colourless gas on
heating. The residual white powder is dissolved in water and when the solution is added to alum
solution, a white gelatinous precipitate is obtained. The original solid was :
(a) Sodium carbonate (b) Sodium bicarbonate
(c) Sodium hydroxide (d) Sodium nitrate
Ans. (b)
Sol. NaHCO3 ¾¾
D
® Na2CO3 + H2O + CO2
Na2CO3 solution alkaline
Al3+ + OH– ¾® Al(OH)3 (white gelatinous ppt)
16. Harsha was trying to neutralize phosphoric acid using various bases. Those available were caustic
soda, lime water and hydrated alumina. If Harsha took 1 equivalent of phosphoric acid each time,
what will be the ratio for moles of each of the above bases required for complete neutralization ?
(a) 1 : 1 : 1 (b) 1 : 0.5 : 0.33
(c) 1 : 2 : 3 (d) 1 : 0.33 : 0.5
Ans. (b)
Sol. n-factor No. of moles
Caustic soda - NaOH 1 1
1
Lime water - Ca(OH)2 2
2
1
Hydrated Alumina - Al(OH)3 3
3
1 eq of phosphoric acid = 1 eq. of NaOH
= 1 eq. of lime water = 1 eq. of Al(OH)3
1 1
1: :
2 3
17. A flask containing SO2 gas was weighed at a particular temperature and pressure. The flask was then
flushed and filled with oxygen gas at the same temperature and pressure. The weight of SO2 in the
flask will be about :
(a) same as that of oxygen (b) one-fourth that of oxygen
(c) four times that of oxygen (d) twice that of oxygen
Ans. (d)
w1 w 2 w1 w 2
Sol. = =
m1 m 2 61 32
w1 = 2w2
(SO2) (CO2)

4
18. Arun needs 1.71 g of cane sugar (C12H22O11) to sweeten his tea. What would be the number of carbon
atoms consumed through sugar in the tea ?
(a) 3.66 × 1022 (b) 7.2 × 1021 (c) 5 × 1021 (d) 6.6 × 1022
Ans. (a)
1.71
Sol. × 12 × 6.023 × 1023 = 3.61 × 1022
342
19. Choose the correct sets which represent the oxides as Acidic : basic : neutral : amphoteric respectively
(i) CO2 : MgO : N2O : H2O (ii) SO2 : NO : CO : Al2O3
(iii) P2O5 : ZnO : NO : Al2O3 (iv) SO3 : CaO : N2O : PbO
(a) i and ii (b) ii and iii
(c) iii and iv (d) i and iv
Ans. (d)
Sol. (i) CO2 - Acidic
MgO - Basic
N2O - Neutral
H2O - Amphoteric
(iv)SO3 - Acidic
CaO - Basic
N2O - Neutral
PbO - Amphoteric
20. During a meteorite shower a few meteorites fell into a water body having pH around 7. The pH of the
water body was measured after meteorite shower and found to be
(a) > 7
(b) < 7
(c) = 7
(d) no change in pH of water due to the meteorite shower
Ans. (a)
Sol. Meteorites are mainly composed of metal oxides hence solution formed is basic.
pH > 7
21. The position of two blocks at successive 0.20-second time intervals are represented by the numbered
squares in the figure below. The blocks are moving towards right.

Block a 1 2 3 4 5 6 7

Block b
1 2 3 4 5

The acceleration of the blocks are related as follows :


(a) acceleration of ‘a’ is greater than acceleration of ‘b’.
(b) acceleration of ‘a’ equals acceleration of ‘b’. Both accelerations are greater than zero.
(c) acceleration of ‘b’ is greater than acceleration of ‘a’
(d) acceleration of ‘a’ equals acceleration of ‘b’. Both acceleration are zero
Ans. (d)
Sol. Velocity is constant for the blocks a and b, so acceleration will be zero.

5
22. In rural areas, an indigenous way of keeping kitchen materials cool is to put them in a box and wrap
the box with weiblanket; the blanket is kept wet as tap is allowed to drip in to its comer. Choose the
correct statement:
(a) This method works because the water from the tap is cold. If one uses room temperatufe water, it
will not work.
(b) Method will work only if the box is a bad Conductor of heat. If one uses tin box, it will not work.
(c) Method doesn't work
(d) method works because the latent heat necessary for evaporation of water in the blanket is taken
from the box so the box and its content remain cool'
Ans. (d)
Sol. A wet blanket is wraped around a box so latent heat necessary for evaporation of water is the blanket
is taken from the box so the box and its content remain cool.
23. In the adjacent circuit what is the current flowing from N to K?
30W 60W
A B J K
30W 10W
G H 60W L
D C N M
30W 60W

120 V
(a) 3A (b) 2A (c) 1A (d) 0.5 A
Ans. (c)
30W 60W

30W 10W 60W


N K
Sol. 30W 60W

V=120 V
10W 10W 20W

V=120 V
120
i= = 3A
40
\ in NK current will be 1A.
24. If x, v and t represent displacement (m), velocity (m/s) and time 9s) respectively for a certain particle
then which pair of the following figures can be best correlated to each other.
x x x x

t t t t
Fig.I Fig.II Fig.III Fig.IV
(a) I & II (b) I & III (c) I & IV (d) None

6
Ans. (b)
Sol. Slope of x - t graph gives velocity
Slope = v = tan q
if q will greater than v will be more.
25. The take off speed of Airbus A340 is 288 km/hr. From the taxi track it comes to the main runway and
waits for a while for the final clearance from Air Traffic control. The aircraft then achieves this speed
within 50 seconds. Neglecting the effect of the wind direction and friction, what should be the minimum
length of main runway decided by civil engineeis for this aircraft for a take-off?
(a) 1800 m (b) 2000 m (c) 2200 m (d) 2400 m
Ans. (b)
5
Sol. u = 0, v = 288 × = 80 m/s
18
80 - 0 8
a= = m/s2
50 5
8
u = 0, v = 80 m/s, a = m/s2, s = ?, t = 50 sec
5
1 1 8
s= × at2 = × = 2500 = 2000 m
2 2 5
26. An empty office chair is at rest on a floor. Consider the following forces :
I- A downward force of gravity,
II-An upward force exerted by the floor,
III-A net downward force exerted by the air.
Then, which of the force(s) is (are) acting on the office chair?
(a) I only
(b) I and II
(c) I, II and III
(d) None of the forces. (Since the chair is at rest there are no forces acting upon it.)
Ans. (b)
N
Sol.

mg
But in air net force will be upward byount force.
27. The ability of eye to focus both near and distant objects, by adjusting its focal length, is called
(a) Myopia (b) Presbyopia
(c) accommodation of eye (d) Tyndall effect
Ans. (c)
Sol. Change in focal length of eye is known as accomodation of eye.
28. In bringing a a-particle, the electrostatic potential energy of the system _________.
(a) increases (b) decreases (c) remains unchanged (d) becomes zero

7
Ans. (a)
Sol. When a particle are brought together electrostatic potential energy increase.
29. A magnet is placed between two coils AB and CD as shown. It is being moved in the direction as
shown by the arrow, then which of the following statement is correct.
(a) looking from end A, current in coil AB will be anticlockwise and looking from end D, the direction
of current in coil CD will be anticlockwise. A B C D
(b) looking from end A, current in coil AB will N S
be clockwise and looking frojm end D, the
direction of current in coil CD will be
G G
clockwise
(c) looking from end A, curent in coil AB will be clockwise and looking from end D, then direction of
current in coil CD will be anticlockwise
(d) looking from end A, current in coil AB will be anticlockwise and looking from end D, the direction
of current in coil CD will be clockwise
Ans. (a)
Sol. Using Lenz's law.
30. A boy throws a steel ball straight up. Consider the motion of the ball only after it has left the boy's
hand but before it touches the ground and assume that forces exerted by the air are negligible. For
these conditions, the force(s) acting on the ball is (are) :
(a) a downward force of gravity along with a steadily decreasing upward force.
(b) a steadily decreasing upward force from the moment it leaves the boy's hand until it reaches its
height point; on the way down there is a steadily increasing downward force of gravity as the
object gets closer to the earth.
(c) constant downward force of gravity along with un upward force that steadily decreases until the
ball reaches its highest point; on the way down there is only a constant downward force of gravity.
(d) constant downward force of gravity only.
Ans. (d)
Sol. In free fall only gravity acts.
31. A large truck collides heat-on with a small compact car. During the collision :
(a) the truck exerts a greater force on the car than the car exertes on the truck.
(b) the car exerts a greater force on the truck than the truck exerts on the car.
(c) the truck exerts a force on the car but the car does not exert a force on the truck.
(d) the truck exerts the same force on the car as the car exerts on the truck.
Ans. (d)
Sol. According to Newton's IIIrd law for every action there is always equal and opposite reaction.
32. A common hydrometer has a uniform scale and its stem is graduated downwards from 0 to 20. While
floating in water, it read 0 and while floating in a liquid of density 1.40 g/cm3, it reads 20. Then the
density of the liquid in which it will reacd 10 is
(a) 0.7 g/cm3 (b) 0.85 g/cm3
(c) 1.17 g/cm3 (d) 2.8 g/cm3
Ans. (c)

8
Sol.

x 20 10

rl = 1.4 rw x x
r

Weight of body = A (20 + x) × rw in water.


Weight of body = A × x × 1.4 in liquid.
Equating A × x × 1.4 = A × (20 + x) × rw
20 + x = 1.4x Þ 0.4 x = 20
x = 50
Similarly in unknown liquid
Þ A × (10 + x) × r = A × x × 1.4
(10 + x) × r = 1.4 × x
60r = 1.4 × 50

1.4 ´ 50 7
r= = = 1.17 g/cm3
60 6
33. For the same angle of incidence, the angle of refraction in three different media A, B, C are 15°, 25°
and 35° respectively. Then which statement is correct : (µA is refractive index of A)
(a) µA is maximum and velocity of light is maximum in medium A.
(b) µA is minimum and velocity of light is maximum in medium A.
(c) µA is maximum and velocity of light is minimum in medium A.
(d) µA is minimum and velocity of light is minimum in medium A.
Ans. (c)
Sol. As we known by Snell's law
A, B, C have r = 15°, 25°, 35° respectively
sin i × 1 = µ sin r
sin i = constant for all. i
air
1
µµ (Q sine is increasing function)
sin r r
Hence as sin r increase µ decreases
\ µA > µB > µC

c 1
and µ = Þµµ
v v
vA < vB < vC.

9
34. A liquid, whose density doesn't change during the motion, is flowing steadily through a pipe of
varying cross sectional area as shown in the adjacent figure. If a1, a2 are the cross sectional areas, v1 ,
v2 are the values of velocities (or speeds) at L and H respectively, then the correct relation between a1,
a2 and v1, v2 is :
H

(a) a1v2 = a2v1 (b) a1v1 = a2v2 (c) a12v2 = a23v1 (d) a1 v12 = a2v22
Ans. (b)
Sol. Equation of continuity states that for steady flow A1V1 = A2V2
35. As shown is adjacent figure two plane mirros. M1 and M2 are inclined to each other at the angle 70°
(angle M1OM2). Incident ray AB makes an algne of incidence q on M1. This ray after relfection at B
on M1 and further at C on M2 travesl along the direction CD, such that path CD is parallel to M1. Then
angle q is _______.
M2

C D
E A

M1
O B
(a) 45° (b) 50° (c) 55º (d) 60°
Ans. (b)
Sol. 70° + 70° + 90° – q = 180° 70°
D

q = 230° – 180° 70°


q
q = 50° 70° 90-q 90-q

36. A copper disc of radius a0 has a hole of radius b0 at the centre, at 0°C. The disce is now heated and
mainteained at 200°C. The new radii of disc and hole are a1 and b1 respectively. for the heated disc it
can be concluded that.

a0

b0

(a) a0 < a1, b0 < b1and density of disc decreases (b) a0 < a1, b0 > b1and density of disc decreases
(c) a0 < a1, b0 < b1and density of disc increases (d) a0 < a1, b0 > b1and density of disc increases
Ans. (a)
Sol. On expansion of Annular disc.
Inner and outer both will expand uniformly
On increase in volume, density decreases
10
37. A concave mirror of radius of curvature 1m is placed at the bottom of a water tahnk. The mirror forms
an image of the sun when it is directly overhead. If the depth of water in the tank is 80 cm, then the
distance of the image formed is _______(refractive index of water is 1.33)
(a) on surface of water (b) 110 cm above mirror
(c) 50 cm above mirror (d) image cannot be formed
Ans. (c)
Sol. R = 100 cm
f = –50 cm From sun u = ¥
u=¥
v=?

1 1 1
= + I 80 cm
f v u

1 1
– = //////
50 v ////////////////////////////

v= –50 cm
38. The equivalent resistance of two resistances in series is 'S'. These resistance are now joined in parallel.
The parallel equivalent resistance is 'P'. If S = n P. Then the minimumpossible value of n is
(a) 2 (b) 3 (c) 4 (d) 5
Ans. (c)
Sol. Let r1 = r2 = r (where r1 and r2 are two resistors)
In series s = r1 + r2 = 2r ...... (1)

r1r2 r2 r
In parallel p= = = ......(2)
r1 + r2 2r 2
equation (1) / equation (2)

s 2r
= =4
p r/2
39. An electron a a-particle enter a regionof uniform magnetic field (of induction B) with equal velocities.
The direction of B is perpendicualr and into the plane of the paper. The qualitatively identify the
direction of paths of electron and the a-particle.
II
I
III

(a) I for a- particle, II for electron (b) I for electron, II for a- particle
(c) I for a- particle, III for electron (d) I for electron, III for a- particle
Ans. (c)
Sol. According to fleming left hand rule we can find force on charge particle.

11
40. Two wave pulses I and II and the same wavelenth. They are travelling in the directions as shown by
the single headed arrows. The resultant sketch of the two wave pulses at some instant of time when P
coincides with R is _________.
I
a

2a

II

a
(a) (b) a

a
a
(c) (d)
Ans. (a)
Sol. Due to superposition principle of wave having same wavelength & frequency, amplitude is simply
added. Logic of standing waves.
41. Ravi mixed two substances A and B in a vessel and left it as it is. After few hours he detected an
alcoholic smell emanating from the vessel. Identify what A and B are:
(a) Salt solution and Lactobacillus (b) Fruit juice and Saccharomyces
(c) Fruit juice and Lactobacillus (d) Salt solution and Saccharomyces
Ans. (b) Fruit juice and Saccharomyces
Sol. Fruit juice ¾¾¾¾¾
Saccharomyces
® Ethanol + CO 2

42. Which amongst the following shows the characters of both plants and animals:
(i) Anabaena (ii) Paramecium (iii) Euglena (iv) Amoeba
(a) i and iv (b) iii (c) ii (d) i and iii
Ans. (b) - (iii)
Sol. Euglena shows mixotrophic nutrition so it shows character of both plants and animals.

43. Which amongst the following are not plastids:


(a) Leucoplasts (b) Chromoplasts (c) Amyloplasts (d) Tonoplasts
Ans. (d) Tonoplasts
Sol. Tonoplast is the outer membrane of plant vacuole. Remaining three are plastids.

44. During a study the number of cells was recorded to increase as follows:
64 ® 128 ® 256 ® 512 ® 1024. This represents:
(a) Budding (b) Meiosis (c) Binary fission (d) Fragmentation
Ans. (c) Binary fission
Sol. In Binary fission each parent cell divides into two daughter cells.

12
45. A plant kept in a box with only a hole for entry of light shows bending, the process called phototropism.
It occurs due to:
(a) Synthesis and diffusion of cytokinin in the leaves
(b) Breakdown of auxin in the shoot
(c) Synthesis and diffusion of abscisic acid
(d) Synthesis and diffusion of auxin in the shoot
Ans. (d) Synthesis and diffusion of auxin in the shoot.
Sol. Auxin accumulates in shaded area of shoot tip, and causes bending of shoot tip towards light i.e.
phototropism.
46. What would be the minimum required length of codon to encode 400 amino acids, if there existed
three purines and pyrimidines each?
(a) 3 (b) 4 (c) 5 (d) 6
Ans. (b) 4
Sol. 3 purines and 3 pyrimidines each so total 6 nitrogen base pairs. We require to encode 400 amino
acids.
6n > 400 Þ 6 × 6 × 6 × 6 = 1296 codons
So, n = 4
Minimum required length of codon would be 4.
47. A 'life- style' disorder among these is:
(a) Hypertension (b) Presbyopia (c) Herpes (d) Scurvy
Ans. (a) Hypertension
Sol. Life style disorders like stress, improper diet, lack of sleep etc. is responsible for causing hypertension.
Presbyopia is due to hardening of lens of eye.
Herpes is caused due to Herpes simplex virus.
Scurvy is caused due to deficiency of Vitamin C.

48. Metamerism is a characteristic of


(a) Hirudinaria (b) Taenia (c) Asterias (d) Pila
Ans. (a) Hirudinaria
Sol. Metamerism is the characteristic feature of phylum Annelids. eg. Hirudinaria (Leech)
49. Health is all about 'eating-fasting' balance. When you fast for extended periods, your cells clean out
and recycle the intracellular garbage. The organelles responsible for this are:
(a) Microtubules (b) Microfilaments
(c) Golgi Apparatus (d) Lysosomes
Ans. (d) Lysosomes
Sol. Lysosomes contain digestive enzymes and help in cleaning of cells.
50. A plant may not exchange CO2 or O2 with air at:
(a) twilight (b) mid-night
(c) late hours in the morning (d) noon
Ans. (a) twilight
Sol. The (light) compensation point is the amount of light intensity where the rate of photosynthesis is
equals to the rate of respiration that is at twilight.

13
51. If a small part of the esophagus of a person is excised, the consequence would be the person will have
to eat _________.
(a) larger portion of food with large time interval
(b) small portions of food at small time interval
(c) small portions of food at large time interval
(d) majorly subsist on liquid diet
Ans. (b) small portions of food at small time interval
Sol. Because small part of esophagus is excised and the patient need to eat small portions of food at small
time interval
52. When heated, the hydrogen bonds between the complementary strands of DNA break and the 2
strands separate in a process called melting. Which of the following pieces of DNA will require maximum
temperature for melting?
(a) 3' AAGGTATACAAT 5' (b) 3' GAGCUAUCCGAG5'
5' TTCCATATGTTA 3' 5'CUCGAUAGGCUC 3'
(c) 3' ACGTCCGCTGCG 5' (d) 3' ATTAGCTAGCAA 5'
5' TGCAGGCGACGC 3' 5' TAATCGATCGTT 3'
Ans. (c) 3' ACGTCCGCTGCG 5',
5' TGCAGGCGACGC 3'
Sol. 3' ACGTCCGCTGCG 5',
5' TGCAGGCGACGC 3'
DNA fragments with more G/C base pairs will require higher temperature for melting. Option (c) has
9 G/C base pairs which is the maximum among all other options.
53. In a self-pollinated plant, what would be minimum number of meiotic divisions required for setting
400 seeds?
(a) 100 (b) 200 (c) 400 (d) 500
Ans. ( d) 500
Sol. For pollen grain uuuuuuuuur 4 microspores.
1 microspore mother cell meiosis

uuuuuuuuur 1 megaspore.
1 megaspore mother cell meiosis
Total number of meiosis = 100 for pollen grains, 400 for megaspore = 500 meioisis to produce 400
seeds.
54. If a flower is producing a large number of minute and smooth pollen, the agency for cross pollination
is most likely to be:
(a) Air (b) Water (c) Insects (d) Bats
Ans. (a) Air
Sol. Wind pollinated pollen should be small is size, light in weight, large in number and non sticky.

55. To meet the increasing demand for food, there have been several 'revolutions'. Which of the following
revolutions is likely to have contributed most to global warming?
(a) Green (b) White (c) Blue (d) Silver
Ans. (b) White
Sol. White revolution has major time contributed to increases milk production but the live stock produces
numerous polluting gases specially methane which is a potent gas for global warming.

14
56. A mammal adapted to desert conditions is likely to have large:
(a) Nostrils (b) Pinnae (c) Muzzle (d) Nails
Ans. (b) Pinnae
Sol. The large ears of the desert animals are used in cooling and dissipating heat.
57. Which of the following feature indicates omnivorous feeding of human species?
(a) Presence of canines as well as premolars and molars
(b) Presence of appendix
(c) Presence of 11th and 12th pair of ribs
(d) Presence of opposable thumb
Ans. (a) Presence of canines as well as premolars and molars
Sol. Presence of canines indicates carnivorous nature and presence of both pre molars and molars are
indicative of hebivorous nature so humans has both the teeth present so that is suggestive of omnivorous
nature.
58. In a dihybrid cross, what is the proportion of organisms with dihybrid geonotype?
(a) 2/16 (b) 6/16 (c) 4/16 (d) 9/16
Ans. (c) 4/16
Sol. In a dihybrid cross the proportion of organism with dihybrid genotype (RrYy) is 4/16.
59. If the cell is using less oxygen molecules than the molecules of carbon dioxide evolved in respiration,
the substrate for respiration has to be:
(a) simple sugars (b) organic acids
(c) fatty acids (d) cholesterol
Ans. (b) organic acids
Sol. Respiratory quotient (RQ) is the ratio of volume of CO2 produced to the volume of O2 consumed in
respiration.
RQ > 1 of organic acids. RQ < 1 of fats
RQ = 1 of simple sugars.
60. Panting is a means of thermoregulation in dogs. This is due to:
(a) high specific heat of water (b) high vapour pressure of water
(c) high latent heat of vapourization (d) high specific gravity of water
Ans. (c)
Sol. Panting is a means of thermoregulation in dogs due to high latent heat of vaporization.
61. How many four digit numbers are there such that when they are divided by 101, they have 99 as
remainder ?
(a) 90 (b) 98 (c) 100 (d) 101
Ans. (a)
Sol. Number is of the form 101 K + 99
Then first four digit number = 101 × 9 + 99 = 1008
Last four digit number = 101 × 98 + 99 = 9997
So number of such four digit numbers is = 90

62. If x = ( )
21 – 20 and y = ( )
18 – 17 , then

(a) x = y (b) x < y (c) x > y (d) x + y = 0


Ans. (b)

15
Sol. x = 21 – 20 , y = 18 – 17

( 21 - 20 )( 21 + 20 ) 1
Þ x =
21 + 20 21 + 20
1
Similarly, y =
18 + 17
1 1
Þ x= ,y=
21 + 20 18 + 17
Þ x<y
63. What is the sum of all odd numbers between 500 and 600 ?
(a) 26000 (b) 27000 (c) 27500 (d) 29500
Ans. (c)
Sol. Odd numbers between 500 and 600 are 501, 503,......... 599.
So number of odd numbers are 50
50
\ Sum of all odd number = (501 + 599)
2
= 25 × 1100 = 27500

1 1 1 1 1 1
64. 1 + 1 + 1 + 1 + 1 + ....................+ 1 = ___________.
2 6 12 20 30 380
(a) 20.25 (b) 20.05 (c) 19.95 (d) 19.85
Ans. (c)

æ1 1 1 1 ö
Sol. [1 + 1 + 1+.................. +1 (19 terms] + ç + + + ............. +
è 2 6 12 380 ÷ø

æ 1 1 1 1 1 1 1 1 1 ö
= 19 + ç 1 - + - + - + - + .......... - ÷
è 2 2 3 3 4 4 5 19 20 ø

æ 1 ö
= 19 + ç 1 - ÷
è 20 ø

19
= 19 + = 19 .95
20
65. A train is running at a speed of 54 km/hr. It is not stopping at a certain station. It crosses the person
showing green flag in 20 seconds and crosses the platform in 36 seconds. What is the length of the
train ?
(a) 240 m (b) 300 m (c) 320 m (d) 360 m
Ans. (b)
Sol. Let length of train = 'L' metre
5
Also, speed of train = 54 × = 15 m/s
18
L
then, 20 = Þ 300 = L
15
Þ L = 300 m

16
66. In triangle ABC, segment AD, segment BE and segment CF are altitudes. If AB × AC = 172.8 cm2 and
BE × CF = 108.3 cm2 then AD × BC = ________.
(a) 136.8 cm2 (b) 132.4 cm2 (c) 129.2 cm2 (d) 128.6 cm2
Ans. (a)
Sol. Given : AB × AC 172.8 cm2, BE × CF = 108.3 cm2
1 1 1
Area of triangle ABC =AD × BC = AC × BE = AB × CF A
2 2 2
AD × BC = AC × BE = AB × CF
Þ (AD × BC)2 = (AC × BE) × (AB × CF)
Þ (AD × BC)2 = (AB × AC) (BE × CF) F E
Þ (AD × BC)2 = 172.8 × 108.3
Þ (AD × BC)2 = 18714.24
B C
Þ AD × BC = 136.8 cm2 D
67. Diagonals of a quadrilateral bisect each other. Therefore the quadrilateral must be a ______.
(a) parallelogram (b) rhombus (c) rectangle (d) square
Ans. (a)
Sol. If diagonals of a quadrilateral bisects each other, then it is a parallelogram.
68. If (a + b + c + d) = 4, then

1 1 1 1
+ + + = ____________.
(1 - a)(1 - b)(1 - c) (1 - b)(1 - c)(1 - d) (1 - c)(1 - d)(1 - a) (1 - d)(1 - a)(1 - b)

(a) 0 (b) 0.25 (c) 1 (d) 4


Ans. (a)
1 1 1 1
Sol. + + +
(1 - a)(1 - b)(1 - c) (1 - b)(1 - c)(1 - d) (1 - c)(1 - d)(1 - a) (1 - d)(1 - a)(1 - b)

1- d +1- a +1- b +1- c


=
(1 - a)(1 - b)(1 - c)(1 - d)

4 - (a + b + c + d)
=
(1 - a)(1 - b)(1 - c)(1 - d)

4-4
= =0
(1 - a)(1 - b)(1 - c)(1 - d)
69. The sum of two numbers is 13 and the sum of their cubes is 1066. Find the product of those two
numbers.
(a) 26 (b) 27 (c) 28 (d) 29
Ans. (d)
Sol. Let the two numbers be are x and y
Then x + y = 13 and x3 + y3 = 1066
Þ x3 + y3 + 3xy (x + y) = 2197
Þ 1066 + 3xy (13) = 2197
Þ 39 xy = 1131
Þ xy = 29

17
70. By which smallest number we should divide 198396198 to get a perfect square ?
(a) 14 (b) 18 (c) 22 (d) 28
Ans. (c)
Sol. Here, 198396198 = 2 × 32 × 72 × 113 × 132
So, it should be divided by 22 to make it a perfect square.
71. What will be the remainder if the number (7)2017 is divided by 25?
(a) 1 (b) 7 (c) 18 (d) 24
Ans. (b)

(7)2017 (72 )1008 (7)


Sol. =
25 25

(50 - 1)1008 (7)1


=
25

(50k + 1)7
= (By Binomial expansion)
25

50k ´ 7 + 7
=
25
Þ Remainder = 7
72. If ABCD is a cyclic quadrilatteral AB = 204, BC = 104, CD = 195, DA = 85 and
BD = 221, then AC = ________.
(a) 210 (b) 220 (c) 225 (d) 240
Ans. (b)
Sol. Let AC = x
By Ptolemy's theorem

195 C
D
x
85 104
221

A 204 B

Þ AB × CD + AD × BC = AC × BD
Þ 195 × 204 + 85 × 104 = 221 × x
Þ 39780 + 8840 = 221x
Þ 48620 = 221x
x = 220 m
73. If x2 + xy + xz = 135, y2 + yz + xy = 351 and z2 + xz + yz = 243, then x2 + y2 + z2 = _________.
(a) 225 (b) 250 (c) 275 (d) 300
Ans. (c)

18
Sol. x2 + xy + xz = 135 ...(1)
y2 + yz + xy = 351 ...(2)
z2 + xz + yz = 243 ...(3)
On adding the above equations we get
x2 + y2 + z2 + 2xy + 2yz + 2xz = 729
(x + y + z)2 = 729
x + y + z = ±27
By eqn (1), x (x + y + z) = 135
135
Þx=± = ±5
27
By eqn. (2), y (y + z + x) = 351
Þ y = ±13
243
By eqn. (3), z = = ±9
27
so x2 + y2 + z2 = 25 +169 + 81 = 275
74. What is the radius of the circumcircle of a triangle whose sides are 30 cm, 36 cm and 30 cm
(a) 15 cm (b) 16 cm (c) 17 cm (d) 18 cm
Ans. (NA)
Sol. Let a = 30 cm , b= 36 cm and c = 30 cm and R be radius of circumcircle of triangle.
abc
Here, R =
4D
30 + 36 + 30
Now, D = s(s - a)(s - b)(s - c) , where s (Semi perimeter of triangle) = = 48
2 A
= 48(48 - 30)(48 - 36)(48 - 30)

= 48 ´18 ´12 ´18 30 30


D = 18 × 12 × 2 = 432 cm O
R
30 ´ 36 ´ 30 B C
36
Then R =
4 ´ 432
R = 18.75 cm
75. On seventy first 'Independence day' there are Tuesday. After how many years there will be Tuesday on
'Independence day' ?
(a) 4 yrs. (b) 5 yrs (c) 6 yrs. (d) 7 yrs.
Ans. (c)
Sol. 71st 'Independence day' 15 Aug 2017
Year Odd days
2017 ® 1
2018 ® 1
2019 ® 1
2020 ® 2
2021 ® 1
2022 ® 1
Now total odd days 7 = zero odd day, so
Calender will repeat in 2023 i.e. after 6 years.

19
76. If p + q + r = 2, p2 + q2 + r2 = 30 and pqr = 10, the value of (1 – p) (1 – q) (1 – r) will be ________.
(a) –18 (b) –24 (c) –27 (d) –35
Ans. (c)
Sol. Given p + q + r = 2, p2 + q2 + r2 = 30, pqr = 10
Now, (1 – p) (1– q) (1– r) = (1 – p –q+ pq) (1 – r)
= (1 – r – p + pr – q + qr – pqr + pq)
= (1 – (p + q + r) + pr + qr + pq – pqr) ....(1)
Also (p + q + r)2 = p 2 + q2 + r2 + 2pq + 2qr + 2rp
(2)2 = 30 + 2 (pq + qr + rp)
Þ pq + qr + rp = –13 ... (2)
From (1) and (2)
Þ (1 – p) (1– q) (1– r)
= [(1 – 2 + (–13) – 10)]
= 1 – 2 – 13 – 10
= – 24
77. The mean of the following frequency distribution is ___________.

Class interval 0 - 10 10 -20 20 -30 30 - 40 40 - 50

Frequency 4 6 8 10 12

(a) 25 (b) 28 (c) 30 (d) 32


Ans. (c)

Sol. Class Interval Frequency xi fi x i


0 - 10 4 5 20
10 - 20 6 15 90
20 - 30 8 25 200
30 - 40 10 35 350
40 - 50 12 45 540

Sfi xi = 1200, Sfi = 40


Sfi x i 1200
Mean = Sf = = 30
i 40
x 2 - bx m - 1
78. If the roots of the equation = are equal and of opposite signs, then the value of 'm' is
ax - c m + 1
_______.

a-b a+b ab a+b


(a) (b) (c) (d)
a+b a-b a+b ab
Ans. (a)
Sol. (x2 – bx) (m + 1) = (m–1) (ax – c)
(m + 1) x2 – b (m + 1) x = a (m – 1) x – c (m – 1)
(m + 1)x2 – [bm + b + am – a] x + c (m – 1) = 0
Then sum of roots should be zero
bm + b + am – a = 0
a-b
m=
a+b

20
æ 1ö æ 3 1 ö æ 2 1 ö æ 1ö
79. If ç x + ÷ = 5, then ç x + 3 ÷ – 5 ç x + 2 ÷ + ç x + ÷ = _________.
è xø è x ø è x ø è xø

(a) 0 (b) 5 (c) –5 (d) 10


Ans. (a)
1
Sol. x + =5
x

1 æ 2 1 ö æ 1ö
Þ x3 + –5 çx + 2 ÷ +çx + x ÷
x3 è x ø è ø
3 2
æ 1ö æ 1ö æ 1ö æ 1ö
Þ ç x + ÷ – 3 ç x + ÷ – 5 ç x + ÷ + 5.2 + ç x + ÷
è xø è xø è xø è xø
3 2
æ 1ö æ 1ö æ 1ö
Þ ç x + ÷ – 2 ç x + ÷ – 5 ç x + ÷ + 5.2
è xø è xø è xø
Þ 53 – 2.5 – 5.52 + 5.2
Þ 0
80. If x2 – 3x + 2 is a factor of x4 – px2 + q, then p, q are
(a) 2,3 (b) 4, 5 (c) 5,4 (d) 0, 0
Ans. (c)
Sol. x2 – 3x + 2 = (x – 2) (x – 1)
Put x = 1 in x4 – px2 + q then
1–p+q=0
p–q=1 ...(1)
Put x = 2 in x4 – px2 + q
16 – 4p + q = 0
4p – q = 16 ...(2)
from (1) and (2)
4p – q = 16
4p – 4q = 4
– +
3q = 12
q = 4, p = 5

21
NSEJS-2018 (IJSO STAGE-I)
Date of Examination : 18th November, 2018
PAPER CODE - JS511

SOLUTIONS
1. A tiny ball of mass m is initially at rest at height H above a cake of uniform thickness h. At some
moment the particle falls freely, touches the cake surface and then penetrates in it at such a constant
rate that its speed becomes zero on just reaching the ground (bottom of the cake). Speed of the ball at
the instant it touches the cake surface and its retardation inside the cake are respectively
æH ö æH ö
(a) 2gh and g ç - 1 ÷ (b) 2g(H - h) and g ç - 1 ÷
èh ø èh ø

æh ö æh ö
(c) 2gh and g ç - 1 ÷ (d) 2g(H - h) and g ç - 1 ÷
èH ø èH ø
Ans. (b)
A ball
Sol.
H–h
H
B
h
C
For A to B
U = 0 m/s
a = g m/s2
S=H–h
V = VB
By third equation v2 = u2 + 2as
2 2
vB = (0) + 2 × g × (H – h)
vB = 2g(H - h)
For B to C
u = vB = 2g(H - h)
S=h
v = vC = 0 m/s
a = –a (retardation)
By third equation
v2 = u2 – 2as
02 = u2 – 2as
v 2B 2g(H - h)
a = u / 2S =
2 =
2´ h 2h

æH ö
= g ç - 1÷
èh ø

1
2. Two sound waves in air have wavelengths differing by 2 m at a certain temperature T. Their notes
have musical interval 1.4. Period of the lower pitch note is 20 ms. Then, speed of sound in air at this
temperature (T) is
(a) 350 m/s (b) 342 m/s (c) 333 m/s (d) 330 m/s
Ans. (a)
Sol. Given that
TL = 20 ms

1 1
fL = T = 20 ´ 10-3 Hz ... (1)
L

Given that

fH
fL = 1.4

fH = 1.4 × fL ... (2)


Now,
lL – lH = 2

V V
- =2
fL fH

é1 1ù
V êf - f ú = 2
ë L Hû

é1 1 ù
V ê f - 1.4f ú = 2
ë L L û

Vé 1 ù
1- =2
fL ë 1.4 úû
ê

V é 0.4 ù
=2
fL êë 1.4 úû

V 14
fL = 2 ×
4

1
V = 7fL = 7 × = 3.5 × 100 = 350 m/s
20 ´ 10 -3

2
3. Two plane mirrors M1 & M2 have their reflecting faces inclined at q. Mirror M1 receives a ray AB,
reflects in at B and sends it as BC. It is now reflected by mirror M2 along CD, as shown in the figure.
Total angular deviation d suffered by the incident ray AB is

M1
D
A

B
d1

q d2 M2
O
C

(a) d = 90° + 2q (b) d = 180° + 2q (c) d = 270° – 2q (d) d = 360° – 2q


M1
Ans. (d)
Sol. d = d1 + d2
B a
= 2a + 2b a
a
= 2(a + b)

= 2(180 – q) q b b M2

d = 360 – 2q C b

4. In the adjacent figure, line AB is parallel to screen S. A linear obstacle PQ between the two is also parallel
to both. AB, PQ and Screen S are coplanar. A point source is carried from A to B, along the line AB.
What will happen to the size of the shadow of PQ (cast due to the point source) on the screen S?

A
P

Q
B

(a) It will first increase and then decrease


(b) It will first decrease and the increase.
(c) It will be of the same size for any position of the point source on the line AB.
(d) Umbra will increase and penumbra will decrease till central position.
Ans. (c)

3
Sol. DCPT ~ DCMR
A M

MR CR
= P
PT CT
C a R
b T
CR D
MR = × PT = PT
CT d d Q

Similarly B N
D
D
RN = QT
d
length of shadow

D D
MR + RN = PT + QT
d d

D D
= (PT + QT) = PQ = constant
d d
T M
A
If point sources is at point A
P x
x D
=
PT d

D Q R
x= PT d
y
d B
D y–x
D
y= QT
d
length of shadow is

D N
RN = y – x = (QT - PT)
d

D
= PQ = constant
d
5. Two particles P 1 and P 2 move towards origin O, along X and Y-axes at constant speed u1 and u2
respectively as shown in the figure. At t = 0, the particles P1 and P2 are at distances a and b respectively
from O. Then the instantaneous distance s between the two particles is given by the relation.
O
P1
u1

u2

P2

(a) s = [a2 + b2 + (u12 + u22)t2 – 2t(au1 + bu2)]1/2 (b) s = [a2 + b2 + (u12 + u22)t2 – 2t(bu1 + au2)]1/2
(c) s = [a2 + b2 + (u12 + u22)t2 + 2t(au1 + bu2)]1/2 (d) s = [a2 – b2 + (u12 + u22)t2 – 2t(au1 + bu2)]1/2
Ans. (a)

4
a
Sol. a – u1t
u1 O
P1
P'1
u1t b – u2t
b
P'2
u2t

P2

after t sec the distance

P '1 P '2 = (OP '1 ) 2 + (OP '2 )2 = (a - u1t) 2 + (b - u 2 t) 2

= a 2 + b2 + t 2 (u12 + u 22 ) - 2t(au 1 + bu 2 )

6. An electric generator consume some oil fuel and generates output of 25 kW. Calorific value (amount
of heat released per unit mass) of the oil fuel is 17200 kcal/kg and efficiency (output to input ratio) of
the generator is 0.25. Then, mass of the fuel consumed per hour and electric energy generated per ton
of fuel burnt are respectively
(a) 0.5 kg, 20000 kWh (b) 0.5 kg, 5000 kWh
(c) 5 kg, 5000 kWh (d) 5 kg, 20000 kWh
Ans. (c)
Sol. (a) h = 0.25
P0 = 25 kW

P0 25
So Pi = = = 100kW
h 0.25
So Ei = Pi × t = 100 kW × 1hr
= 100 kWh = 100 × 3.6 × 106 J

J
Calorific value of fuel is = 17200 × 103 × 4.2
kg
So amount of fuel required in 1 hr.

Ei 100 ´ 3.6 ´ 10 6
m= =
Calorific value 17200 ´ 10 3 ´ 4.2
= 4.983 kg
; 5 kg
(b) Energy contained per ton of fuel
= 17200 × 103 × 4.2 × 1000 J
E0 = h × Ei = 0.25 × 17200 × 103 × 4.2 × 1000
= 1.806 × 1010 J

1.806 ´ 1010
= kWh
3.6 ´ 106
= 5016.66 kWh
; 5000 kWh

5
7. Image is obtained on a screen by keeping an object at 25 cm and at 40 cm in front of a concave mirror.
Image in the former case is four times bigger than in the latter. Focal length of the mirror must be
____.
(a) 12 cm (b) 20 cm (c) 24 cm (d) 36 cm
Ans. (b)
f
Sol. m =
f -U

f f
m1 = =
f - ( -25) f + 25

f
m2 =
f + 40
but m1 = 4 × m2
f f
= 4´
f + 25 f + 40
f + 40f = 4f2 + 100f
2

3f2 = –60 f
3f = –60
f = –20 c.m.
8. A glass cube of refractive index 1.5 and edge 1 cm has a tiny black spot at its center. A circular dark
sheet is to be kept symmetrically on the top surface so that the central spot is not visible form the top.

1 1 1
Minimum radius of the circular sheet should be (Given : = 0.707, = 0.577, = 0.447)
2 3 3

(a) 0.994 cm (b) 0.447 cm (c) 0.553 cm (d) 0.577 cm


Ans. (b)
n a AB
Sol. By snell's law sin ic = =
n g OB
A r B

n a AB 1 r iC
0.5cm

Þ = Þ = iC
n g OB æ3ö r + (0.5)2
2
ç ÷
è2ø 1 cm
O

2 r
Þ =
3 r + (0.5) 2
2

4 r2
Þ =
9 r 2 + (0.5)2
Þ 4r2 + 4 × (0.5)2 = 9r2
4r2 + 4 × 0.25 = 9r2
5r2 = 1
1
r= = 0.447 cm
5

6
9. A metal rod of length L at Temperature I, when heated to temperature T', expands to new length L'.
These quantities are related as L' = L(1 + a[T' – T] where a is a constant for that material and called as
coefficient of linear expansion. Correct SI unit of a is
(a) m-K–1 (b) m-K (c) K–1 (d) a is a pure number
Ans. (c)
Sol. L1 = L [1 + a DT]
1
By principle of homogeneity [a] =
[ DT]
unit of a is k–1
10. A paramedical staff nurse improvises a second's pendulum (time period 2s) by fixing one end of a
string of length L to a ceiling and the other end to heavy object of negligible size. Withing 60 oscillations
of this pendulum, she finds that the pulse of a wounded soldier beats 110 times. A symptom of
bradycardia is pulse < 60 per minute and that of tachycardia is > 100 per minute. Then the length of
the string is nearly _______ and soldier has symptoms of ______.
(a) 1 m, bradycardia (b) 4 m, bradycardia (c) 1 m, tachycardia (d) 4 m, tachycardia
Ans. (a)

l
Sol. T = 2p =2
g

l g 10
Þ p2 × g = 1 Þ l = 2 ; ; 1m
p 10
If time period of pulse beat is T1 then.
60 × 2 = 110 × T1
120 12
T1 = =
110 11
1 11
So Frequency of pulsebeat f1= = Hz
T1 12

11
So number of beats in 1 minute = ´ 60 = 55 beats/min.
12
So soldier has symptoms of bardycardia
11. Each resistance in the adjacent circuit is R W. In order to have an integral value for equivalent resistance
between A and B, the minimum value of R must be :

A B

(a) 4 W (b) 8 W (c) 16 W (d) 29 W

7
Ans. (b)

R R/2 R
R R
R R/2 R
R R
Sol. A B
R R/2 R
R
R/2 R
R R

R 5R/4 R
R R
A B
R 5R/4 R

R 13R/8 R
A B
29R/8
A B

29R
RAB =
8
In order to have an integral value for RAB.
R=8W
th
æ3ö
12. A block of wood floats on water with ç ÷ of its volume above water. It is now made to float on a salt
è8ø
solution of relative density 1.12. The fraction of its volume that remains above the salt solution now,
is nearly _______.
(a) 0.33 (b) 0.44 (c) 0.67 (d) 0.56
Ans. (b)
Sol. While floating on water

æ5 ö
FB1 = rw ´ v w ´ g = rw ´ ç v s ÷ ´ g ....(1)
è8 ø
while floating on salt water
FB2 = rsw ´ v sw ´ g = (1.12 × rw) × vsw × g ....(2)

But FB1 = FB2

5
rw × × vs × g = 1.12 × rw × vsw × g
8

5
Þ vsw = × vs
8 ´ 1.12

vsw
Þ v = 0.558
s

So fraction of volume above the salt water = 1 – 0.558 = 0.44

8
13. Suppose our scientific community had chosen force, speed and time as the fundamental mechanical
quantities instead of length, mass and time respectively and they chose the respective units of magnitudes
1
10 N, 100 m/s and s. Then the unit of mass in their system is equivalent to _______ in our system.
100
(a) 10 kg (b) 10–3 kg (c) 10 kg (d) 10–1 kg
Ans. (b)
Sol. [M] µ [F]a [V]b [T]c
[M] µ [MLT–2]a [LT–1]b [T]c
[M] µ [M]a [L]a + b [T]–2a – b + c
By comparison
a=1; a+b=0; –2a – b + c = 0
b = –a c = 2a + b
b = –1 =2–1=1
Substituting the values of a, b and c
F´t 10 ´ 1
So m = = = 10–3 kg
v 100 ´ 100
14. Two equally charged identical pith balls are suspended by identical massless strings as shown in the
adjacent figure. If this set up in on Mercury (g = 3.7 m/s2), Earth (g = 9.8 m/s2) and Jupiter (g = 24.5 m/s2),
then angle 2q will be _______.

q q

1.0 m 1.0 m

+ +
Q Q

(a) maximum on Mercury


(b) maximum on Earth, as it has atmosphere
(c) maximum on Jupiter
(d) the same on any planet as Coulomb force is independent of gravity
Ans. (a)

Sol.

q q
1.0 m 1.0 m
Tcosq Tcosq

Fe Fe
Tsinq Tsinq

mg mg

9
By F.B.D.
T cosq = mg ....(1)
T sinq = Fe .....(2)
Fe
tanq =
mg
1
tanq µ
g
So value of q will be maximum on mercury.
15. Three objects of the same material coloured white, blue and black can withstand temperature up to
2000°C. All these are heated to 1500°C and viewed in dark. Which option is correct?
(a) White object will appear brightest
(b) Blue object will appear brightest
(c) Black object will appear brightest
(d) Being at the same temperature, all look equally bright
Ans. (c)
Sol. Black will appear brightest at high temperature. Because in thermal equilibrium black body emits
more energy than any other body.
16. A car running with a velocity of 30 m/s reaches midway between two vertical parallel walls separated
by 360 m, when the driver sounds the horn for a moment. Speed of sound in air is 330 m/s. After
blowing horn, the first three echoes will be heard by the driver respectively at ________.
(a) 1.2 s, 2.4 s, 3.0 s (b) 1.0 s, 2.4 s, 3.0 s (c) 1.0 s, 2.0 s, 3.0 s (d) 1.2 s, 2.4 s, 3.6 s
Ans. (b)
Sol. For first echo

v = 30 m/s

180 m

30t + 330t = 360


360 × t = 360
t = 1 sec

For second echo

v = 30 m/s

30 m
vs

210

330t – 30 × t = 2 × 210
300t = 420
t = 1.4 sec.
So total time = 1 + 1.4 = 2.4 sec.

10
For third echo

v = 30 m/s

72 m

vs

108
30t + 330 t = 2 × 108
360 t = 216
216
t= = 0.6 sec.
360
So total time = 2.4 + 0.6 = 3.0 sec.
17. Choose correct option from the following statements from electrostatics :
(I) If two copper spheres of same radii, one hollow and the other solid are charged to the same
electrical potential, the solid sphere will have more charge.
(II) A charged body can attract another uncharged body.
(III) Electrical lines of force originating from like charges will exert a lateral force on each other, while
those originating from opposite charges can intersect each other.
(a) Only (I) is correct. (b) Only (II) is correct.
(c) Only (I) and (II) are correct. (d) All (I), (II) and (III) are correct.
Ans. (b)
Sol. Only (II) is correct
Charge resides only on the outer surface of the conductor. Since radius is same and charge will be
same to have the same potential.
Electrical line of force cannot intersect.
18. Refer the adjacent circuit. The voltmeter reads 117 V and ammeter reads 0.13 A. If the resistance of
voltmeter and ammeter are 9 kW and 0.015 W respectively, the value of R is _____.

A
R

(a) 500 W (b) 1 kW (c) 1.5 kW (d) 2 kW


Ans. (b)
9000 W
Sol. V

A
A R B

Þ VAB = i × Req

é R ´ 9000 ù
117 = 0.13 × ê ú
ë R + 9000 û
R = 1000 W

11
19. A bar magnet is allowed to fall freely from the same height towards a current carrying loop along its
axis, as shown in the four situations I to IV. Arrows show direction of conventional current. Choose
the situations in which the potential energy of the magnet coil interaction is maximum _____.

I N II S III N IV S

S N S N

(a) I, III (b) I, IV (c) II, IV (d) II, III


Ans. (b)
Sol. In (I) & (IV) magnet has to come against the magnetic force (repulsive force). So P.E. will be maximum.
20. A beaker is completely filled with water at 4°C. Consider the following statements:
(I) Water will overflow if the beaker is cooled for some time.
(II) Water will overflow if the beaker is heated for some time.
Select correct option regarding (I) and (II).
(a) Only (I) is correct (b) Only (II) is correct
(c) Both (I) and (II) are correct (d) Neither (I) nor (II) is correct
Ans. (c)
Sol. At 4ºC on cooling or heating water expands so
Both I and II are correct.
21. P3– has a larger radius than atom of P because
(a) There is greater coulombic attraction between the nucleus and electrons in the p3– ion.
(b) The core electrons in p3– exert a weaker shielding force than those of a neutral atom.
(c) The nuclear charge is weaker in p3– than it is in P.
(d) The electrons in p3– have a greater coulombic repulsion than those in P atom.
Ans. (d)
Sol. The electrons in P3– have a greater coulombic repulsion than those in P atom
22. A substance is dissolved in water, forming a 0.5 molar solution. If 4.0 L of solution contains 240 g of
the substance, what is the molecular mass of the substance?
(a) 60 g/mole (b) l20 g/mole (c) 240g/mole (d) 480 g/mole
Ans. (b)
Sol. 120 g / mole

240 1
0.5 = ´ Þ MM = 120 g
MM 4

12
23. A car battery was kept for charging and after getting fully charged density of the battery acid (H2SO4)
was measured and found to be 1.28 g cm–3. If initial molarity of battery acid was 4.2 M then mass
percentage will be around.
(a) 28% (b) 30% (c) 32% (d) 34%
Ans. (c)
%mass ´ 1.28 ´ 10
Sol. 4.2 =
98
Þ % mass = 32.15%
24. Element "X" with atomic mass 10 was allowed to react completely with element "Y" of atomic mass
20 to form a compound. When this compound was analysed it was found that it contains 60% of X
and 40% of Y by weight. The simplest formula of this compound will be
(a) X3 Y (b) x2Y3 (c) Y3 x (d) x6Y4
Ans. (a)
Sol. X 3 Y
X Y
60% 40%
10 20
6 2
3 1
X3 Y
25. 4.095 X 1024 nitrogen atoms are filled in an enclosed gas cylinder of capacity two litre. The number of
moles of nitrogen gas in the cylinder is ____.
(a) 14.7 (b) 6.8 (c) 3.4 (d) 2.9
Ans. (c)
Sol. 4.095 × 1024 N-atoms = 2.0475 × 1024 N2-molecules
2.0475 ´ 1024
= 3.39
6.023 ´ 1023
26. When surface tension experiment with capillary tube is performed, water rises up to 0.1 m. If the
experiment is carried out in space, water will rise in capillary tube _____.
(a) up to height of 0.1 m (b) up to height of 0.2 m
(c) up to height of 0.98 m (d) along its full length
Ans. (d)
Sol. along its full length due to absence of gravity and under effect of adhesive forces whole surface will
be covered.
27. Deepa was studying properties of gases. She took a flask and filled it with sulphur dioxide gas, and
weighed it at temperature T and pressure P. The weight of the flask containing the gas was found to be
W1 . She then flushed the flask, cleaned and filled it with methane at the same temperature and pressure.
The weight of the flask containing oxygen was found to be W2. She repeated the process with oxygen
under the same conditions and found the weight to be W3 : The ratio of the weights W1 : W2 : W3 is
(a) 2 : 1 : 4 (b) 4 : 2 : 1 (c) 4 : l : 2 (d) 1 : 2 : 4
Ans. (c)
Sol. Underline oxygen in above question was consider as methane, taking this correction answer as follows...
4:1:2
SO2 CH 4 : O2
W1 = n × 64 : W2 = n × 16 W3 = n × 32 (Avogadro Law)
4 : 1 : 2

13
28. Four gas jars filled with sulphur dioxide gas were inverted into troughs of water by four students P, Q,
R, S. The following observations and inference were reported by them.
P : Water did not enter the gas jar and sulphur dioxide is soluble in water.
Q : Water rushed into the gas jar and sulphur dioxide is soluble in water.
R : Water did not enter in the gas jar and sulphur dioxide is insoluble in water.
S : A small amount of water entered the gas jar slowly and sulphur dioxide is sparingly soluble in
water.
Then the correct set of observations and inference is reported by,
(a) P (b) Q (c) R (d) s
Ans. (d)
Sol. S : As the SO2 gas dissolves in water, will enter into the flask because SO2 is less soluble.
29. A solution of pure aluminium sulphate containing 0.170 g of aluminium ions is treated with excess of
barium hydroxide solution. Total weight of the precipitate will be:
(a) 0.5 g (b) 2.7 g (c) 1.7 g (d) 0.54 g
Ans. (b)
Sol. Al2 (SO4 )3 + 3Ba(OH)2 ® 3BaSO4 (ppt) + 2Al(OH)3 (ppt)
342 g Excess 3 × 233
1.0766 g 2.20 g 0.49 g

formed by
2.6910 g
0.17 g Al3+
30. A region of one square meter area was given to each Suhas, Bobby, Sandy and Kimi in a garden. The
daffodil plants grow best in the soil having a pH range of 6.0 to 6.5. If the soil has a pH 4.5, to grow
daffodils, Suhas added common salt, Bobby added sodium phosphate, Sandy added aluminium
sulphate and Kimi added ammonium chloride in their allotted area. Who was successful in growing
daffodil?
(a) Suhas (b) Bobby (c) Sandy (d) Kimi
Ans. (b)
Sol. Salt of strong base and weak acid will undergo anionic hydrolysis result the increase in soil pH
PO4 + H2O ƒ HPO4 + OH–
3– 2–

31. Electrons in the last shell of X, Y, W and Z are 2, 6, 4 and 1 respectively. Which of the following
statement is correct?
(a) melting point of compound formed by X and Y is more than that of by W and Z.
(b) compound formed by X and Y is more volatile than that of by W and Z.
(c) melting point of compound formed by X and Z is more than that of by W and Y.
(d) Incomplete information so inference cannot be drawn.
Ans. (a)
Sol. Melting point of compound formed by X and Y is ionic in nature whereas W and Z will be co-valent
in nature, hence earlier exhibit higher m.p.

14
32. W g of pure coal was combusted in pure dry oxygen. The carbon dioxide gas obtained was absorbed
in 0.1 M KOH solution. The complete absorption of CO2 required 5 cm3 of 0.1 M KOH. The amount
of coal combusted is
(a) 3 mg (b) 6 mg (c) 11 mg (d) 12 mg
Ans. (a)
Sol. Number of milimoles of KOH = 2 × Number of milimoles of CO2
Þ 5 × 0.1 = 2 × Number of milimoles CO2
5 ´ 0.1
\ moles of CO2 = = 2.5 × 10–4
1000 ´ 2
\ moles of Coal = 2.5 × 10–4
\ mass of Coal = 2.5 × 10–4 × 12 g
= 0.003 g
= 3 mg
33. Sulphur di-oxide gas and ammonia gas were mixed in different proportions. The pair of gases containing
same number of molecules at NTP is __________.
(a) 1120 cm3 of SO2 + 0.85 g of ammonia (b) 0.25 g mole of SO2 + 2240 cm3 of ammonia
(c) 1680 cm3 of SO2 + 1.7 g of ammonia (d) 0.25 g mole of SO2 + 0.85 g of ammonia
Ans. (a)
1120
Sol. Moles of SO2 = = 0.05 moles
22400
molecules of SO2 = 0.05 NA
0.85
moles of NH3 = = 0.05 moles
17
molecules of NH3 = 0.05 NA
34. A strip of iron with mass 15.5 g is placed in a solution containing 21.0 g copper sulphate. After some
time the reaction stops. Iron strip was found to have mass 8.5 g. The mass of copper formed was
found to be 8.60 g. Find the mass of ferrous sulphate formed in this reaction.
(a) 19.40 g (b) 18.40 g (c) 17.40 g (d) 16.40 g
Ans. (a)
Sol. Fe + CuSO4 ® FeSO4 + Cu
t = 0 15.5 g 21.0 g (LR) 0 0
t = t 8.5 g 0 36.5 – 8.6 8.6 g
– 8.5
= 19.4 g
Law of mass conservation.
35. Sonu has N/2 HCl solution and Monu has N/10 HCl solution. They are asked to prepare 2 litres of
N/5 HCl solution. What volume of two solutions be mixed?
(a) (0.5 + 1.5) litre (b) (1.0 + 1.0) litre (c) (0.3 + 1.7) litre (d) (0.2 + 1.8) titre
Ans. (a)
Sol. (0.5 + 1.5) litre

æN ö éN ù N
ç 2 ´ x ÷ + ê10 ´ (2 - x) ú = 5 ´ 2
è ø ë û
x = 0.5 litre

15
36. A solution (P) was prepared by dissolving 6.3 g of oxalic acid in 100 ml water. 25 ml of this solution
was taken and was further diluted to 250 ml to prepare solution (Q). What weight of NaOH in ppm will
be required to neutralize 10 ml of solution (Q)?
(a) 10 ppm (b) 20 ppm (c) 40 ppm (d) 80 ppm
Ans. (c)
Sol. oxalic acid
6.3
g / ml oxalic acid initial solution
100
6.3
Þ ´ 25 g oxalic acid taken
100
6.3 25
Þ ´ g / ml oxalic acid solution taken
100 250
6.3 25
´ ´ 10 g oxalic acid taken for reaction
100 250
Þ COOH . 2H2O + 2NaOH ® COONa + 4H2O
| |
COOH COONa
Q 126 g oxalic acid reacts = 2 × 40 g NaOH
6.3 25 2 ´ 40 6.3 25
\ ´ ´ 10 g oxalic reacts = ´ ´ ´ 10 g = 40 mg
100 250 126 100 250
37. Which of the following can improve the quality of petrol?
(a) n heptane (b) benzene (c) n hexadecane (d) iso-octane
Ans. (d)
Sol. Iso-octane
38. 2KBrO3 + 12H+ + l0e– ® Br2 + 6H2O + 2K+
From above reaction the equivalent weight of KBrO3 can be calculated as (M is molecular weight of
KBrO3 )
(a) M/5 (b) M/10 (c) M/12 (d) M/2
Ans. (a)
Sol. 2KBrO3 ® Br 2
+5 O
nƒ = 5
M
Equivalent Weight =
5
39. Shaila took about l0 cm3 of a diluted Potassium hydrogen carbonate solution in a test tube. To this
solution she added few drops of universal indicator. The colour of the solution turned:
(a) orange (b) geen (c) blue (d) yellow
Ans. (c)
Sol. Blue
K+HCO3 + H2O ƒ K+ + H2CO3 + OH–1
–1

40. Which of the following is incorrect?


(a) Chalcocite – Copper (b) Magnetite – Iron
(c) Calamine – Aluminium (d) Galena – Lead
Ans. (c)
Sol. Calamine : ZnCO3

16
41. Let AB be a diameter of circle C1 of radius 30 cm and with center O. Two circes C2 and C3 of raddi 15
cm and 10 cm touch C1 internally at A and B respectively. A fourth circle C4 touches C1,C2 and C3 .
What is the largest possible radius of C4?
(a) 12 cm (b) 15 cm (c) 20 cm (d) 30 cm
Ans. (c)
Sol. There are two cases,
Case 1 :

C1

C2 C3

O2 10 O1 O3
A 10 B
10 15 15
–r

10 15
30

r
r r
O4 C
4

We have O1O4 = 30 – r
O4
Now, consider triangle O2O3O4
O2 O1 DO1O2 O 4
We have O O = DO O O 10+r 15+r
1 3 1 3 3
30-r
20 30 ´ 10 ´ r1 (20 - r)
using heron's formula = O2
20 O1 15
O3
15 30 ´ 15 ´ r1 (15 - r)

4 2(20 - r)
Þ =
3 3(15 - r)
16 40 - 2r
Þ =
9 45 - 3r
Þ 720 – 48 r = 360 – 18r
Þ 30 r = 360
Þ r = 12
C1

B C3
10 C4
10 C2
Case 2 : 10
15
15 A

Diameter of C4 is 10 + 15 + 15 = 40
Þ R = 20 cm
Therefore, maximum radius is 20 cm.

17
42. A 5 × 5 × 5 cube is built using unit cubes. How many different cuboids (that differ in at least one unit
cube) can be formed using the same number of unit cubes?
(a) 1000 (b) 1728 (c) 2730 (d) 3375
Ans. (d)
Sol. In order to make cuboid we need two lines out of 6 lines from each dimensions
6
Hence, C2 ´ 6 C 2 ´ 6 C 2 = 3375

43. What is the largest value of the positive integer k such that k divides n2(n2 – 1) (n2 – n – 2) for every
natural number n?
(a) 6 (b) 12 (c) 24 (d) 48
Ans. (d)
Sol. n2(n2 – 1)(n2 – n – 2)
n2(n + 1)(n – 1)(n + 1)(n – 2)
= (n – 2)(n – 1)(n)2(n + 1)

= (n – 2)(n4–1)(n)(n +3
1) (n)(n+1)
1444 24444 1424 3
4 consective termsarealwaysdivsible by twoconsective terms
8as wellas3, hence by 24 alwaysdivisible by 2

So, overall number is divisible by 24 × 2 = 48


44. A person kept rolling a regular (six faced) die until one of the numbers appeared third time on the top.
This happened in 12th throw and the sum of all the numbers in 12 throws was 46. Which number
appeared least number of times?
(a) 6 (b) 4 (c) 2 (d) l
Ans. (c), (d)
Sol. 3x1 + 2(x2 + x3 + x4 + x5) + x6 = 46
For x6 least ® x6 = 1 (say)
3x1 + 2(x2 + x3 + x4 + x5) = 45
Now, x1 should be odd and largest x1 = 5
2(x2 + x3 + x4 + x5) = 30
x2 + x3 + x4 + x5 = 15
Now remaining number 2, 3, 4, 6, 2 + 3 + 4 + 6 = 15, also
For x6 least ® x6 = 2 (say)
3x1 + 2(x2 + x3 + x4 + x5) = 44
Now, x1 should be even and largest x1 = 6
2(x2 + x3 + x4 + x5) = 26
x2 + x3 + x4 + x5 = 13
Now remaining number 1, 3, 4, 5, 1 + 3 + 4 + 5 = 13
So, both 1 as well as 2 is satisfying the given condition.
45. In a square ABCD, a point P is inside the square such that ABP is an equilateral triangle. The segment
AP cuts the diagonal BD in E. Suppose AE = 2. The area of ABCD

(a) 4 + 2 3 (b) 5 + 2 3 (c) 4 + 4 3 (d) 5 + 4 3


Ans. (a)

18
D C
Sol.
P
60°
E 105°
2 75°

15
°
60° 45°
A a B
using sine rule in DAEB
a 2
=
sin 75° sin 45°
2 ´ sin 75° 3 +1
a= = 2´ ´ 2 = 3 + 1 = side of square
sin 45° 2 2
a= 3 +1
Area = a2 = ( 3 + 1)2 = 4 + 2 3
46. Let n be a positive integer not divisible by 6. Suppose n has 6 positive divisors. The number of
positive divisors of 9n is
(a) 54 (b) 36 (c) 18 (d) 12
Ans. (c), (d)
Sol. Let n be of the form ab2 or a5, Then number of factor of 9n i.e, 9ab2 or 9a5
For 9ab2 ,
If a = 3, b ¹ 0 (mod 2) Þ 33b2, number of factors = (3 + 1)(2 + 1) = 4 × 3 = 12
If b = 3, a ¹ 0 (mod 2) Þ 34a, number of factors = (4 + 1)(1 + 1) = 5 × 2 = 10
For 9a5 ,
If a = 3 Þ 9·35 = 37, number of factors = (7 + 1) = 8
If a = 3, a ¹ 0 (mod 3) Þ 32a5, number of factors = (2 + 1)(5 + 1) = 3 × 6 = 18

a+x - a-x 2ab


47. The value of when x = is
a+x + a-x 2
b2 + 1
(a) a (b) b (c) x (d) 0
Ans. (b)
a+x - a-x ( a + x - a - x )2 1
Sol. = ´ (Rationalising)
a+x + a -x ( a + x + a - x) ( a + x - a - x)
(a + x) + (a - x) - 2 a 2 - x 2 (2a - 2 a 2 - x2 ) a - a 2 - x 2
= = =
(a + x) - (a - x) 2x x
2ab
Now, put x = 2
b +1
4a 2 b2
a - a2 -
(b2 + 1)2
Þ
2ab
b2 + 1
a(b2 + 1) - a 2 (b 2 + 1)2 - 4a 2 b 2
=
2ab

19
(b2 + 1) - (b2 - 1)2 b2 + 1 ± (b 2 - 1)
= =
2b 2b
1
= b or
b
2a
Note : In paper code JS511 & JS513, there are printing mistakes. 'x' is given as x = 2
.
b +1
For JS511 & JS513, it should be given as Bonus.
2ab
This solution is made considering x = , given in JS512 & JS514
b2 + 1
48. Two regular polygons of different number of sides are taken. In one of them, its sides are coloured red
and diagonals are coloured green; in the other, sides are coloured green and diagonals are coloured
red. Suppose there are 103 red lines and 80 green lines. The total number of sides the two polygons
together have is:
(a) 23 (b) 28 (c) 33 (d) 38
Ans. (b)
Sol. Let the two polygon have number of sides n1n2 respectively, then number of diagonals in two polygon are

n1 (n1 - 3) n (n - 3)
and 2 2 respectively
2 2
so, by given condition in problem we have
n 2 (n 2 - 3)
n1 + = 103
2
Þ 2n1 + n22 – 3n2 = 206 ......(1)
n1 (n1 - 3)
n2 + = 80
2
Þ 2n2 + n12 – 3n1 = 160 ...... (2)
equation (1) – (2)
Þ 2(n1 – n2) + (n22 – n21) – 3(n2 – n1) = 46
Þ (n2 – n1)(n1 + n2 – 5) = 46 = 2 × 23 = 23 × 2
Now, we have two cases
n2 – n1 = 2
n1 + n2 – 5 = 23
Þ n1 + n2 = 28 Þ possible case
or n2 – n1 = 23
n1 + n2 – 5 = 2
n1 + n2 = 7 Þ not possible
\ Total number of sides of two polygon together = 28
49. A box contains some red and some yellow balls. If one red ball is removed, one seventh of the
remaining balls would be red; if one yellow ball is removed, one-sixth of the remaining balls would be
red. If n denotes the total number of balls in the box, then the sum of the digits of n is
(a) 6 (b) 7 (c) 8 (d) 9
Ans. (b)

20
Sol. Let R and Y be the number of Red and Yellow balls respectively is the box.
Then, according to the question,
Y + R -1
=R–1
7
Þ Y + R – 1 = 7R – 7
Þ 6R – Y = 6 .....(1)
R + Y -1
Also, =R
6
Þ R + Y – 1 = 6R
Þ Y – 5R = 1 .....(2)
6R – 1 – 5R = 6
ÞR=7
\ Y = 36
Hence, number of balls = 7 + 36 = 43
so sum of digits of n = 7
50. Let ABCD be a rectangle. Let X and Y be points respectively on AB and CD such that AX : XB = 1 : 2 =
CY : YD. Join AY and CX ; let BY intersect CX in K ; let DX intersect AY in L. If m/n denotes the ratio
of the area of XKYL to that of ABCD, then m + n equals
(a) 9 (b) 11 (c) 13 (d) 15
Ans. (b)

[XLYK] m
Sol. =
[ABCD] n
A a X 2a B
DALX ~ DYLD

DY h 2 h 2 h1
= Þ 2 =
AX h1 h1 1 h2

\ h2 = 2b L

h1 = b K

Area of rectangle ABCD = 9ab


ArDADY = 3ab h2
h1
3ab D 2a y a C
ArDBYC =
2
ab
ArDAXL =
2
ArDBXK = 2ab
3ab ab
\ XLYK Area = 9ab –(3ab + + + 2ab) = 2ab
b
2 2
= 2ls
[XLYK] 2ab 2
\ = =
[ABCD] 9ab 9
m 2
\ =
n 9
m + n = 11

21
51. Let ABC be an equilateral triangle. The bisector of ÐBAC meets the circumcircle of ABC in D. Suppose
DB + DC = 4. The diameter of the circumcircle of ABC is
(a) 4 (b) 3 3 (c) 2 3 (d) 2
Ans. (a)
Sol. Given, DB + DC = 4 A
Since, AD is angle bisector
Þ BD = DC = 2 30°
we have ÐDBE = ÐDAC = 30°
\ ÐABD = 60° + 30° = 90°
BD 60°
Now, In DABD, sin30° = E
AD B 30° C

1 2 60°
Þ =
2 AD
Þ AD = 4 cm D
\ diameter = 4
52. Let Tk denpte the k-th term of an arithmetic progression. Suppose there are positive integers m ¹ n
such that Tm = 1/n and Tn = 1/m. Then Tmn equals
1 1 1
(a) (b) + (c) 1 (d) 0
mn m n
Ans. (c)
Sol. Let the first term of AP be a and the common difference be d.
1
Then, Tm = a + (m – 1)d = ....(1)
n
1
Tn = a + (n – 1) d = ....(2)
m
equation (1) – (2)
1 1
(m – n)d = -
n m
(m - n)
(m – n)d = ....(3)
mn
1
Put d = in equation (1)
mn
(m - 1) 1
a+ =
mn n
1 (m - 1) m - m +1 1
a= - = = .....(4)
n mn mn mn
Now, Tmn = a + (mn – 1)d
1 (mn - 1) 1
= + (Q a = d = )
mn mn mn

1 + mn - 1
=
mn
Tmn = 1

22
53. In a triangle ABC, let AD be the median from A; let E be a point on AD such that AE : ED = 1 : 2 ; and
let BE extended meets AC in F. The ratio of AF/FC is
(a) 1/6 (b) 1/5 (c) l/4 (d) 1/3
Ans. (c)
Sol. By menelaus theorem we have
A
BC DE AF
× × =1
BD EA FC 1 F
( BEF transverse) E E
2 2 AE 2
´ ´ =1
1 1 FC 2x 2x
B D C
AF 1
Þ =
FC 4
Aliter :
D(AFE) AF
ar =
D(EFC) FC

D(AFB) AF DABE + DAEF


Also, ar = =
D(BFC) CB DBED + DDEC + DECF
By ratio theroem
DAFB - DAFE AF
=
DBFC – DEFC FC

DAEB AF x 1
= = =
DBEC FC 4x 4
54. If sinq and cosq are roots of the equation px2 + qx + r = 0, then:
(a) p2 – q2 + 2pr = 0 (b) (p – r)2 + q2 – r2
(c) p2 + q2 – 2pr = 0 (d) (p – r)2 = q2 + r2
Ans. (a)
Sol. px2 + qx + r = 0

q
sinq + cosq = -
p

r
sinq × cosq =
p
(sinq + cosq)2 = sin2q + cos2q + 2sinqcosq
2
æ qö r
ç- ÷ = 1+ 2
è pø p

q 2 2r
Þ - =1
p2 p
Þ q2 – 2rp = p2
Þ p2 – q2 + 2pr = 0

23
55. For a regular k-sided polygon, let a(k) denotes its interior angle. Suppose n > 4 is such that a(n – 2),
a(n), a(n + 3) forms an arithmetic progression. The sum of digits of n is
(a) 2 (b) 3 (c) 4 (d) 5
Ans. (b)
Sol. For any n-sided regular polygon,

(n - 2) ´ 180°
Interior angle a(n) =
n
Now, a(n – 2), a(n), a(n + 3) are in A.P

(n - 4) ´ 180° (n - 2) ´ 180° (n + 1) ´ 180°


Þ , , are in AP
n-2 n n +3

2(n - 2) n - 4 n + 1
Þ = +
n n -2 n +3

2n - 4 n 2 - n - 12 + n 2 - n - 2
Þ =
n n2 + n - 6
Þ (2n – 4)(n2 + n – 6) = n(2n2 – 2n – 14)
Þ 2n3 – 2n2 – 16n + 24 = 2n3 – 2n2 – 14n
Þ 2n = 24
Þ n = 12
Q sum of the digits of n = 3
56. The sum of 5 numbers in geometric progression is 24. The sum of their reciprocals is 6. The product
of the terms of the geometric progression is
(a) 36 (b) 32 (c) 24 (d) 18
Ans. (b)

a a
Sol. Let the terms of GP be , , a, ar, ar2
r2 r
Where a is the first term and r is the common ratio

a a
ATQ, + + a + ar + ar2 = 24
r2 r

æ1 1 ö
a ç 2 + + 1 + r + r 2 ÷ = 24
è r r ø

1 1 24
2
+ + 1 + r + r2 = .....(1)
r r a

r2 r 1 1 1
Also, + + + + 2 =6
a a a ar ar

1æ 2 1 1 ö
= ç r + r +1+ + 2 ÷=6
aè r r ø

24
1 24
= ´ =6 (from (1))
a a

24
=6
a2

24
a2 =
6
a2 = 4
a = ±2
a a
Product of terms : ´ × a × ar × ar
r2 r
= a5
= (±2)5
= ±32
only 32 is given in options

57. Digits a and be are such that the product 4a1´ 25b is divisible by (in base 10). The number of ordered
pairs (a, b) is
(a) 15 (b) 8 (c) 6 (d) 4
Ans. (Bonus)
Sol. Number is : 4a1 ´ 25b
36 = 22 × 32
Hence we have to check for divisibility by 4 and 9.
Product of two numbers, A × B is divisible by 4 and 9, if
Case 1 : A is devisible by 36 and B can be any number,
Case 2 : B is divisible by 36 and A can be any number,
Case 3 : A is divisible by 4 and B is divisible by 9,
Case 4 : A is divisible by 9 and B is divisible by 4,
Case 5 : A is divisible by 2 and B is divisible by 18,
Case 6 : A is divisible by 18 and A is divisible by 2,
Case 7 : A is divisible by 3 and B is divisible by 12,
Case 8 : A is divisible by 12 and B is divisible by 3,
Case 9 : A is divisible by 6 and B is also divisible by 6.
Case 1, Case 3, Case 5, Case 6, Case 8, Case 9 are neglected because 4a1 is odd and hence cannot
be divisible by any even number.
Case 2 : 25b , 5b must be divisible by 4, so be can be 2 or 6 only.
2 + 5 + b = 7 + b must be divisible by 9.
for b = 2, 7 + 2 = 9 is divisible by 9.
for b = 6, 7 + 6 = 13 not divisible by 9.
So b = 2, a can be any digit from {0, 1 ......9}
So (a, b) = (0, 2), (1, 2), (2, 2) ....... (9, 2) Þ total 10 pairs.

25
Case 4 : 4a1 : 4 + a + 1 = 5 + a must be divisible by 9.
so a can be 4 only.

25b : 5b must be divisible by 4.


So b can be 2 or 6 only.
So (a, b) can be (4, 2) and (4, 6)
But (4, 2) is already counted in case (2)
So (4, 6) ....... 1 Pair

Case 7 : 4a1 : 4 + a + 1 = 5 + a must be divisible by 3, So a can be 1, 4, 7.

25b : 5b must be divisible by 4 so b can be 2 or 6, also 2 + 5 + b = 7 + b must be divisible by 3, b can


be 2, 5, 8. Hence be can be 2 only.
So (a, b) can be (1, 2), (4, 2), (7, 2) But all of the these have correctly been considered.
Overall total 10 + 1 = 11 Pairs will be there.

58. The integer closest to 111...1 - 222...2 , where there are 2018 ones and 1009 twos, is

101009 - 1 101009 - 1 102018 - 1 102018 - 1


(a) (b) (c) (d)
3 9 3 9
Ans. (a)
Sol. Using gemetric progression for sum of 'n' term

102018 - 1
111 ...... 1 + 2018 time =
9

æ 101000 - 1 ö
222 ..... 2 + 1009 have = 2 ´ ç ÷
è 9 ø
Now,
= 111......1 - 222...2

102018 - 1 2 1009
= - (10 - 1)
9 9

1
= 102018 - 2x101009 + 1
3

1 1
= (101009 - 1)2 = (101009 - 1)
3 3
59. In a triangle ABC, a point D on AB is such that AD : AB = 1 : 4 and DE is parallel to BC with E on AC.
Let M and N be the mid points of DE and BC respectively. What is the ratio of the area of the quadrilateral
BNMD to that of triangle ABC?
(a) 1/4 (b)9/32 (c) 7/32 (d) 15/32
Ans. (d)

26
Qud.[BNMD]
Sol. =?
D[ABC] A
Now, since DADE ~ DABC(DE || BC)

D xM x E
[ADE] AD2
\ =
[ABC] AB2
h
2
æ AD ö
=ç ÷ y y
è AB ø B C
N
2
æ1ö
=ç ÷
è4ø

1
=
16

[ADE] 1
Also, =
[ADE] + [DBEC] 16

16[ADE] = [ADE] + [DBEC]


15[ADE] = [DBEC]

[DBEC] 15
\ = ....(1)
ADE 1

[ABC]
and [ADE] = .....(2)
16
Also, let h be the height of trapezium BCED and let DM = ME = x and BN = NC = y

1
\ ar of trapezium [DMNB] = (x + y)h
2

1
and ar of trapezium [CNME] = (x + y)h
2

1
\ [DMNB] = [CNME] = [BCED]
2
or [DBEC] = 2[BNMD]
Put in (1), we get

2[BNMD] 15
=
[ABC] 1
16

32[BNMD] 15 [BNMD] 15
= Þ\ =
[ABC] 1 [ABC] 32

27
é 10 2 ù é102 ù é102 ù é 10 2 ù
60. The number of distinct integers in the collection ê ,
ú ê ,
ú ê ú ,..... ê ú , where [x] denotes the
ë 1 û ë 2 û ë 3 û ë 20 û
largest not exceeding x, is
(a) 20 (b) 18 (c) 17 (d) 15
Ans. (d)

é 102 ù é102 ù é102 ù


Sol. ê ú, ê ú , ..... ê ú
ë 1 û ë 2 û ë 20 û

é 102 ù é 102 ù
ê ú = 100 ê ú= 9
ë 1 û ë 11 û

é 102 ù é 102 ù
ê ú = 50 ê ú= 8
ë 2 û ë 12 û

é 102 ù é 102 ù
ê ú = 33 ê ú= 7
ë 3 û ë 13 û

é 102 ù é 102 ù
ê ú = 25 ê ú= 7
ë 4 û ë 14 û

é 102 ù é 102 ù
ê ú = 20 ê ú =6
ë 5 û ë 15 û

é 102 ù é 10 2 ù
ê ú = 16 ê ú= 6
ë 6 û ë 16 û

é 102 ù é 10 2 ù
ê ú = 14 ê ú= 5
ë 7 û ë 17 û

é 102 ù é 10 2 ù
ê ú = 12 ê ú= 5
ë 8 û ë 18 û

é 102 ù é 10 2 ù
ê ú = 11 ê ú= 5
ë 9 û ë 19 û

é 102 ù é 10 2 ù
ê ú = 10 ê ú= 5
ë 10 û ë 20 û

\ distinct integers = 15

28
61. True coelom is not present in animals of:
(a) Platyhelminthes (b) Annelida
(c) Echinodermata (d) Arthropoda
Ans. (a)
Sol. Platyhelminthes is an example of acoelomate animals.
62. The intracellular organelle that is responsible for formation of acrosomal vesicle is:
(a) Endoplasmic reticulum (b) Golgi apparatus
(c) Mitochondrion (d) None of the above
Ans. (b)
Sol. Golgi apparatus
Acrosomal vesicle form during spermiogenesis by golgibody.
63. The genetically modified (GM) brinjal in India has been developed for:
(a) Enhancing shelf life (b) Insect-resistance
(c) Drought-resistance (d) Enhancing mineral content
Ans. (b)
Sol. The main goal behind genetically modified brinjal is to make it insect resistance.
64. A scientist observed few cells under a microscope with following characters:
i. Cells divided by binary fission or fragmentation, or budding
ii. Cells moved with the help of flagella
iii. Ether lipids were observed in cell membranes
iv. Peptidoglycans were noted in the cell walls
Which of the following category do the cells belong to?
(a) Archaea (b) Plant cells
(c) Unicellular eukaryotes (d) Cyanobacteria
Ans. (a)
Sol. Petidoglycan cell wall present in monera means cyanobacteria, archaea etc. But if we compare
cyanobacteria and archaea flagella absent in cyanobacteria and present in Archaea. According to
question scientist observed cell moved with the help of flagella and flagella absent in cyanobacteria so
answer will be (a).
65. Character(s) of acquired immunity is (are):
(a) Differentiation between self and non-self (c) Retains memory
(b) Specificity of antigen (d) All the above
Ans. (d)
Sol. The immunity acquired by an organism after birth is known as acquired immunity. Which has following
characteristics
(a) It can differentiate between self and non-self antigen.
(b) It is specific for specific antigen.
(c) It retains the memory of first encounter with antigen

29
66. Instead of using chemical fertilizers in a paddy field, a farmer thought of employing nitrogen fixation
technique. Amongst the following which would be beneficial for his cause?
(a) Glycine max - Rhizobium (b) Cycas-Nostoc
(c) Casuarina - Frankia (d) Azolla-Anabaena
Ans. (d)
Sol. Azolla (Fern) and Anabaena (B.G.A.) both show symbiotic relationship and perform N2 fixation. This
type of relationship is beneficial for paddy field. It increases efficiency of nitrogen use and reduce the
water pollution. So Azolla and Anabaena symbiotic relation replace the uses of chemical fertilizers.
So among all the four option the pair of Azolla - Anabaena is suitable to grow in paddy field as Azolla
is a water fern and paddy grows in fields filled with water.
67. An action potential in the nerve fibre is produced when positive and negative on outside and inside of
the axon membrane are reversed because:
(a) all potassium ions leave the axon
(b) more potassium ions enter the axon as compared to sodium ions leaving it
(c) more sodium ions enter the axon as compared to potassium ions leaving it
(d) all sodium ions enter the axon
Ans. (c)
Sol. The action potential in nerve fibre is produced due to more permeability of voltage gated sodium
channels which causes large influx of Na+ which reverses the polarity of axon membrane.
68. A geneticist was studying the pathway of synthesis of an amino acid 'X' in an organism. The presence
(either synthesized de novo or externally added) of 'X' is a must for the survival of that organism. She
isolated several mutants that require 'X' to grow. She tested whether each mutant would grow when
different additives, P, Q, R, S and T were used.'+' indicates growth and '-' indicates the inability to
grow in the mutants tested. Find out the correct sequence of additives in the biosynthetic pathway of
'X'.

Organisms Additives
P Q R S T
Wild-type + + + + +
Mutant 1 – – – – +
Mutant 2 – + + + +
Mutant 3 – – + – +
Mutant 4 – + + +
(a) P ® Q ® R ® S ® T (b) P ® R ® S ® Q ® T
(c) T ® P ® Q ® S ® R (d) P ® S ® Q ® R ® T
Ans. (d)
Sol. '×' is an aminoacid which is required for the growth of organism / Survival.
'+' indicates growth
'–' indicates inability to grow
(So number of '+' increase/number of mutant increase that increase the survival of additives.)
P ® S ® Q ® R ® T

(+) (+, +) (+, +, +) (+, +, +, +) (+, +, +, +, +)

30
69. In a case of mammalian coat color, the principal gene identified is 'C which codes for a tyrosinase
enzyme. In case of rabbits four different phenotypes are observed Full Color > Chinchilla > Himalayan > Albino
(in order of the expression of gene 'C and its alleles). In a progeny obtained after crossing two rabbits,
the percentages of Chinchilla, Himalayan and Albino rabbits were 50, 25 and 25 respectively. What
must have been the genotypes of the parent rabbits?
(a) CchCchX Cchc (b) CchCh X Cchc (c)Cchc X Ch c (d) ChCh X CchCch
Ans. (c)
Sol. Cch ¾® Chinchilla
Cch ¾® Himalayan cc ¾® Albino
Cchc X Chc

CchCh Cchc Chc cc


Chinchilla Chinchilla Himalaya Albino

50 25 25
70. It was observed in a group of tadpoles of a mutant frog reared in a laboratory that their development
was arrested at a particular stage. The exact tissue that was affected by the mutation is unknown. The
development was then resumed and accelerated by injecting the tadpoles with the extracts prepared
from various tissues of the wild type frogs. The observations of the experiment are given below.
Experiment No. Tissue Extract Observations
1 Anterior lobe of pituitary Development resumed
2 Posterior lobe of pituitary Development did not resume
3 Thyroid gland Development resumed
4 Anterior lobe of pituitary + Thyroid gland Development resumed
5 Anterior + posterior lobe of pituitary Development resumed
6 Posterior lobe of pituitary + Thyroid gland Development did not resume
From the above observations, find out the tissue that is affected by the mutation.
(a) Anterior lobe of pituitary (b) Posterior lobe of pituitary
(c) Thyroid gland (d) Both pituitary and thyroid gland
Ans. (a)
Sol. Anteror pituitary gtand has tropic action which stimualate thyroid gland in turns it screte thyroxine,
which stimnlate metamorphosis, mutated Anterior lobe of pituitary would not produc TSH so
development is arrested.
71. Identify the odd ones from each group (A and B) based on same criterion.

Group A Group B
Salmon Alpine salamander
Bullfrog Spiny anteater
Platypus Common toad
Bull shark Crocodile
(a) Platypus, Alpine Salamander (b) Bull shark, Alpine salamander
(c) Bullfrog, Crocodile (d) Platypus, Common toad
Ans. (b)
Sol. The criteria that is used to distinguish members of group A as well members of groups B is ovo-
viviparity.
In group a bull shark is ovo-viviparous while other three members are oviparous.
In group B alpine salamander is ovo-viviparous while other three members are oviparous.
31
72. A patient was administered a chemical agent called Guanfacine hydrochloride after the patient showed
the symptoms like shortness of breath and headache. Guanfacine hydrochloride is a known stimulant
of central a2-adrenergic receptors of the medulla regulating the sympathetic nervous system. The
patient in this case must be suffering from______.
(a) Hypertension (b) Hyperstimulation
(c) Hyperpolarization (d) None of the above
Ans. (a)
Sol. Drug guanfacine hydrochloride is used to treat the cases of hypertension and this drug binds to the
a2 -adrenergic receptors present in medulla oblongata.
73. A bacterial dsDNA molecule, 2988 bp in length, was found to have the following composition:
The respective values of X and Y are:

T C A G
Strand I 348 X 1400
Strand II 650 Y
(a) 1400 and 590 (b) 590 and 1400 (c) 590 and 590 (d) None of the above
Ans. (c)
Sol. According to chargraff's rule

A+T
=1
C+G

(Strand I) T = (Strand II) A = 348


(Strand II) T = (Strand I) A = 650
(Strand I) G = (Strand II) C = 1400
\ Value of Y = total nmber of base in strand II – Total number of T, C, A in strand II
= 2988 – (348 + 650 + 140)
= 590
Value of X = Total nimber of base in strand I – Total number of T, A, G, in strand I
= 2988 – (348 + 650 + 1400)
= 590
74. What would be the length of a polypeptide translated from mRNA which is encoded by 2988 bp of a
bacterial gene?
(a) 989 (b)992 (c)995 (d)998
Ans. (c)
Sol. Number of bases in m-RNA = 2988

2988
Number of codon in m-RNA =
3

(as codon is made up of 3 bases) Þ 996 codons


Out of 996 codon, one codon is stop codon that does not code for any amino acid.
So the length of polypeptide translated will be 995 amino acid.

32
75. A student recorded the data for five types of cells as given below:

Character P P Q R S T
Cell wall + + – – +
Centrioles – – – + _
Chloroplast – + – – _
Mitochondrion – + + + +
Nucleus – + - + +
Plasma membrane + + - + +
RNA/DNA + + + + +
Vacuoles + + - + +

The five cell types P, Q, R, S and T are:


(a) P - Bacterium, Q - Plant, R- Virus, S - Animal, T - Fungus
(b) P - Bacterium, Q - Plant, R- Virus, S - Fungus, T - Animal
(c) P - Fungus, Q - Plant, R- Bacterium, S - Animal, T - Virus
(d) P - Plant, Q - Bacterium, R- Virus, S - Animal, T - Fungus
Ans. (a)
Sol. According to question cell wall present in bacteria, plant and fungus but Nucleus and membrane
bound cell organelles present in both plant and fungus and absent in Bacteria (underdeveloped nucleus)
so accurate order will be
P-Bacterium, Q-Plant, R-Virus, S-Animal, T-Fungus
76. An environment conservation group performed a survey of some diverse locations in the country and
represented it as under:

Which amongst these sites should be included as a biodiversity hotspot?


(a) Site A (b)Sitc B (c)Site C (d)Site D
Ans. (a)
Sol. To qualify as a biodiversity hotspot on myers 2000 edition of the hosrpot-map a region must contain
atleast 1500 species of vasular plants as endonics.

33
77. A bacterium has a generation time of 50 minutes. A culture containing 108 cells per mL is incubated
for 300 minutes. What will be the number of cells after 300 minutes?
(1) 64 × 103 cells (b) 6.4 × 108 cells (c) 64 × 109 cells (d) 6.4 × 109 cells
Ans. (d)
Sol. Generation time = 50 minutes
Initial = 108 cells /ml
Cell count
Incubation time = 300 minutes
[F = I × 2n] (F = final number of bacteria)
(I = Initial number of bacterial = 108)
n = number of generation

300
n= n= =6
50

F = 108 × 26 = 108 × 64 = 6.4 × 109


78. The blood grouping system is an example of 'multiple allelism. In order to find out the gene products
of various gene variants, different enzymes (codes used for the purpose of experimentation are X and
Y) from four blood samples were assayed. The enzymes were quantified and the information obtained
from these experiments is given in percentages in the following table. indicates presence of an enzyme
and indicates the absence of that enzyme from the blood sample. The standard codes for dominant
and recessive alleles are considered. Identify the blood groups of subjects and choose the correct
option of their genotypes from given options. (In table: + means present, – means absent)

Subjects Ramesh Ali Sophia Balwinder

Enzymes P/A % P/A % P/A % P/A %

X + 50 + 50 + 100 – –

Y – – + 50 – – + 100

(a) IAi, ii, IBi, IAIB (b) IAi, IAIB, IAIA, IBIB

(c) IBi, IAIB, ii, IBi (d) IBi, ii, IAIB, IAi
Ans. (b)
Sol. Let X enzyme is coded by IA
Let Y enzyme is coded by IB
In case of Ramesh 50% X is formed which shows that alleles are in heterozygous condition = IAi
In case of Ali 50% X and 50% y is formed which shows the presence of both dominant allels IAIB. In
case of Sofia 100% of × indicate the genotype IAIA
In case of Balwinder 100% of Y indicates the genotyfie IBIB.

34
79. In an experiment, a scientist discovered a darkly stained chromatin body on the periphery of nucleus
of epithelial cells obtained from an eight year old boy. This is indicative of a particular syndrome.
Find out the best possible chromosome combination of their parents from the options given below;
which have the highest probability of producing the child under investigation. 'A' indicates autosome.
'X' and 'Y' represent the sex chromosomes.

(a) 22AA+XY, 22AA+XXX (c) 22AA+XY, 22AA+XX

(b) 22AA+XXY, 22AA+XXX (d) 22AA+XXY, 22AA+XX

Ans. (a)

Sol. In the given options, option (b) and (d) indicates having kleinfelter's syndrome so the male will be
infertile. Option (c) shows normal genotype so the option A has highest probabilty of producing the
child under investigation.

80. A millionaire Mr. Jim, died recently. Two women, Mary and Lou, claiming to have a child by Jim
approached the police demanding a share in his wealth. Fortunately Jim's semen sample was
cryopreserved. The scientists used DNA fingerprinting technique to study the three highly variable
chromosome regions. The results obtained are shown in the adjoining figure:

Marry's Lou's
Jim Mary child Lou child

After studying the DNA profile, which of the alleged heirs are children of Jim?

(a) Mary's child (b) both are children of Jim

(c) Lou's child (d) none are children of Jim

Ans. (b)

Sol. From the given DNA profile it can be observed that the some DNA bands of Jim are matching with
some DNA bands of Mary's Lou's child so we can conclude that both the child's belong to Jim.

35
NSEJS-2019 (IJSO STAGE-I)
Date of Examination : 17th November, 2019
PAPER CODE - 54

SOLUTIONS
1. A group of students was studying development of an organism under controlled laboratory conditions. Following
observations were made by them.
i. The larvae had a rod-like supporting structure that separated the nervous system and the gut.
ii. A prominent central cavity was present in the transverse section of the part of the nervous system of the larvae;
while the adults had cerebral ganglia as the main component of the nervous system.
iii. The eyes were prominently seen in larvae.
iv. The tails were absent in the adults, which the larvae had.
v. A lot of phagocytic activity was observed before conversion of larvae into adults.
vi. The adults had a cuticular exoskeleton.
The organism under study must be belonging to:
(a) Amphibia (b) Pisces (c) Protochordata (d) Arthropoda
Ans. (c)
Sol. All the observations are of protochordata (Protochordata include urochordata and cephalochordata, urochordata
has retrogressive metamorphosis while cephalochordata has progressive metamorphosis)
2. In case of mice coat colour, two genes are responsible for colour of the hair. Gene 'A' is responsible for distribution
of pigments on shaft of hair. Wild type allele of 'A' produces a yellow band on dark hair shaft (agouti), whereas
recessive allele produces no yellow band. There is another allele of A, known as AY, which is embryonic lethal in
homozygous condition only. In an experiment, two yellow mice were crossed to obtain a progeny of 6 pups. What
would be the most probable number of agouti mice among them?
(a) 0 (b) 2 (c) 4 (d) None of the above
Ans. (b)

Sol. Yellow × Yellow

AA
y
× AA
y

y y y y
AA AA AA AA
Yellow In homozygous
agauti living condition dead

Probability of agouti is 1/3


out of 3 ® 1 agouti
so out of 6 ® 2 agouti

1
3. A stain was developed by a group of scientists to stain a particular cell organelle. The stain was tested on various
tissues derived from an autopsy sample from a mammal. The organelles were counted. The results showed maximum
number of the organelles in cells of brain, lesser in cells of heart, least in mature sperms and absent in erythrocytes.
Identify the organelles from following options.
(a) Nissl bodies (b) Mitochondria (c) Golgi bodies (d) Endoplasmic reticulum
Ans. (b)
Sol. The organelle which is obtained after stain is mitochondria which is abudantly present in brain cell lesser in heart,
least in mature sperms and completely absent in erythrocytes.
4. Pinus sylvestris grows at low temperatures in Russia. The plant survives under such freezing conditions due to the
presence of:
(a) Saturated lipids in plasma membrane (b) Glycoproteins in plasma membrane
(c) Glycolipids in plasma membrane (d) Polyunsaturated lipids in plasma membrane*
Ans. (d)
Sol. The plants which grows at low temperature basicaly have unsaturated lipid in their plasma membrane. For maintaining
flexibility of plasma membrane.
5. In an experimental setup, certain pathogen caused a disease in primates with nasal congestion, sore throat and fever
being the common symptoms. The scientists injected an extract from blue-green mold as the first line of action.
However, the symptoms did not subside. The possible causative agents of the disease were listed out as follows.
i. a virus ii. a fungus
iii. a conjugation deficient bacterium iv. a tapeworm
Choose the correct option from the following that indicate the pathogen.
(a) i, ii (b) i, iii (c) ii, iv (d) iii only
Ans. (b)
Sol. Nasal congestion also called sinusitis has sore throat and fever which is caused by virus, conjugation deficient
bacterium has no role in it. (Conjugation in bacteria is a process in which plasmid are transferred by themselves
alone or along with other DNA element from one cell to another cell through conjugation tube)
6. A process is represented in the adjacent figure. The arrows indicate the flow of a R
biochemical reaction. The arrowhead points to the product, while the base of the
arrow indicates the template biomolecule. What do P, Q, R, and S represent?
(a) P : Replication, Q : Translation, R : Transcription, S : Reverse Transcription X Y
P
(b) P : Transcription, Q : Replication, R: Reverse Transcription, S : Translation S
Q
(c) P : Reverse Transcription, Q : Replication, R : Translation, S : Transcription
Z
(d) P : Reverse Transcription, Q : Replication, R : Transcription, S : Translation
Ans. (d)

Transcription
Sol.
Reverse
RNA Transcription DNA

Translation
Replication
Protein

2
7. The whooping cranes were on the verge of extinction with only 21 individuals in wild in 1941. After conservation
measures, the cranes are now included in the endangered category by IUCN. The highlight of the conservation
efforts is the reintroduction of the whooping cranes in wild. This was possible due to raising of the young cranes in
absence of their parents by biologists dressed in crane costumes. Aircraft Guided bird migration technique was used
for teaching the captive-bred cranes to follow the scientists to learn the migratory route. What type of animal
behaviour might be responsible for these captive-bred cranes to follow the crane costume dressed scientists?
(a) Cognitive learning (b) Habituation (c) Operant conditioning (d) Genetic Imprinting
Ans. (d)
Sol. Imprinting : A type of behavior that includes both learned and innate components is called imprinting. it is become
an important component of efforts to save endangered species.
8. A 4 µm long bacterial cell was magnified and drawn to a dimension of 6 cm. How many times has it been
magnified?
(a) 1.5 × 103 (b) 15 × 104 (c) 1.5 × 104 (d) 1.5
Ans. (c)
Sol. 4µm long bacterial cell = 4 × 10–6 m
magnification to a dimension of 6 cm = 6 × 10–2 m

magnifying dimension 6 ´ 10-2


magnification = = = 1.5 × 104
actual dimention 4 ´ 10-6

9. In the baking industry, when the dough is prepared, various ingredients are mixed together with the flour. At one
instance, the dough was fermented, but failed to rise sufficiently during the baking process. Choose the correct
cause(s) from following possibilities.
i. The salt was mixed before the fermentation process was completed
ii. The sugar was added in excess
iii. Yeast granules were not activated prior to mixing with the flour.
(a) i, iii (b) iii only (c) i, ii, iii (d) i, ii
Ans. (a)
Sol. In process of fermentation, if the dough failed to rise sufficiently during backing process, this might be due to
inactivation of yeast. Salt addition to the better can be done immidiately after fermenting.
10. Given below are four statements.
1. Prokaryotic cells are unicellular while eukaryotes are multicellular.
II. Histones are present in eukaryotes and absent in prokaryotes.
III. The nucleoid contains the genetic material in prokaryotes and eukaryotes.
IV. Prokaryotic flagellum is composed of flagellin while eukaryotic flagellum is composed of tubulin.
Identify which amongst these are false.
(a) I and II (b) III and IV (c) II and III (d) I and III
Ans. (d)
Sol. because prokaryotic cell are unicellular but eukaryotic are not only multicellular, it may be unicellular also. Only
prokaryotic genetic material is called nucleoid.

3
11. The students of a college were working on regeneration using Planaria (Platyhelminthes) and Asterias (Echinodermata).
Planaria was cut in three pieces, namely, a piece with head, with tail and the middle piece. Asterias (bearing five
arms) was cut in such a way that after separation, six pieces were obtained, namely, an arm with a portion of the
central disc, four pieces cut from tips of each of the remaining arms and the remaining body. The animals were
allowed to regenerate completely.
How many Planaria and Asterias respectively will be obtained after the completion of regeneration in both?
(a) 1, 1 (b)3,2 (c) 3, 6 (d)l,2
Ans. (b)
Sol. Planaria can be cut into pieces, and each piece can regenerate into a complete organism over the course of a few
weeks.
Most species of sea stars must split part of their central disc along with a limb for regeneration to occur. It is very
unlikely that a severed limb will be able to regenerate into a full-grown starfish unless it is already attached to at least
a portion of the central disc.
12. Fecundity in animal world is the maximum possible ability of an individual to produce offsprings during its entire
lifetime. Following factors were checked for their effect on fecundity of different animal models.
i. Availability of food during breeding season
ii. Mode of fertilization
iii. Population density
Which of these factor(s) can regulate fecundity?
(a) i, ii (b) ii, iii (c) i, ii, iii (d) None of the above
Ans. (c)
Sol. Fecundity means maximum possible ability of an individual to produce offsprings during life time.
Availability of food, mode of fertilization and population density regulate the fecundity.
13. In case of peppered moths, pale and dark moths are observed. Pale variety is known to be the wild type variety.
During industrial revolution, industrial melanism led to prevalence of dark variety around the cities and pale variety
continued to be in majority in areas away from the industries.
After enforcement of regulations for controlling pollution, reappearance of pale moths in majority was observed
around cities again. Driving force(s) for these adaptive changes is/are:
i. Increased pollution around industries
ii. A stable transposition of a gene in moths
iii. Limitations of the vision of birds to differentiate dark moths on darkened barks and pale moths in presence of
lichens
iv. Ability of lichens to grow on barks in less polluted areas only.
(a) i, iv (b) i, iii, iv (c) i, ii (d) i, ii, iii and iv
Ans. (b)
Sol. It is the example of progressive/ directional Natural selection, in which only one extremity is selected, when environment
condtion change it is shited towards another extremity, pale moth is selected after controlling of pollutions and its
reappearance observed in cities again.
14. Four different human body fluid samples were subjected to quantification of hydrogen ion concentration. mEq/L is
the unit of measurement for hydrogen ion concentration. The results of the experiment were as follows:
Sample A: 1.6 X 102 units Sample B: 4.5 X 10–5 units
Sample C: 1 X 10–3 units Sample D: 3 X 10–2 units
Identify the samples in sequence from A to D.

4
(a) Gastric HCl, Venous blood. Intracellular Fluid, Urine
(b) Venous blood, Intracellular Fluid, Gastric HCl, Urine
(c) Urine, Gastric HCl, Venous blood, Intracellular Fluid
(d) Intracellular Fluid, Urine, Gastric HCl, Venous blood
Ans. (a)
Sol. PH = –log10 [H+]
According to this formula correct answer is (a)
15. Any damage or injury to a particular area causes nociceptors to release some chemicals, which carry the signal to
the higher centres in the nervous system for the processing and a subsequent action. However, there is a difference
in the way in which the stimulus is received which is related to the acuity of the detection. Fingertips are more
sensitive as compared to the forearm. Following reasons for the observed phenomenon were suggested.
i. The receptive fields in the fingertip are smaller
ii. The number of nociceptors per receptive field in the forearm is lesser
iii. The amount of prostaglandins released by the nociceptors per receptive field is more in fingertips
The most probable reason(s) for this may be:
(a) i (b) i, iii (c) ii, iii (d) i, ii, iii
Ans. (d)
Sol. Due to any external stimulus (injury) nociceptors release prostaglandins which is more in fingertips per receptive field
that's why fingertips are more sensitive as compared to forearm
16. Rate of photosynthesis in hydrophytes depends on various parameters. The adjacent graph shows the effect of one
parameter (while keeping all the others constant) on the rate of photosynthesis. Rate of photosynthesis is plotted on
Y axis. Identify the parameter which is plotted along X axis:
Rate of Photosynthesys

Parameter
(a) light intensity (b) wavelength (c) temperature (d) CO2 concentration
Ans. (b)
Sol. According to figure wavelength of light affect the photosynthesis.
17. An organism has 27 pairs of homologous chromosomes. In each daughter cell after completion of mitosis and in
each gamete after completion of meiosis II, ________ and ________ chromosomes would be present respectively.
(a) 27 and 27 (b) 54 and 27 (c) 108 and 54 (d) 54 and 108
Ans. (b)
Sol. A cell have 27 pairs of chromosome.
2n = 54 chromosomes.

5
2n = 54

In mitosis : 2n
2n = 54

is II n = 27
ios
Me
I n
sis
eio n = 27
M
In meiosis II : 2n
is II n = 27
= 54 e ios
M
n
n = 27

So ans is 54 and 27 respectively.


18. Gymnosperms are called naked seed bearing plants' because they lack:
(a) Male gamete (b) Ovule (c) Ovary (d) Seeds
Ans. (c)
Sol. Gymnosperm are called ‘naked seed bearing plant’ because they lacks ovary.
19. Rahul sprayed a chemical 'X' on a plant with rosette habit. After few days, he found the internodal distances to have
increased suddenly. The chemical 'X' might be:
(a) Ethylene (b) Abscisic acid (c) Auxin (d) Gibberellic acid
Ans. (d)
Sol. Gibberellic acid initiate the internodal distance.
20. On a study tour, plants with leathery leaves with thick cuticle, vivipary, salt glands, apogeotropic roots, and stomata
limited to abaxial surface were observed. The plants might be:
(a) Bromeliads (b) Cycads (c) Mangroves (d) None of the above
Ans. (c)
Sol. Mangroves plant have the properties of thick cuticle, vivipary, apogeotropic root and so on.
21. An element Y is a white translucent solid at room temperature and exhibits various allotropic forms. Some compounds
of element Y find application in agriculatural industry. Y forms two solid oxides which dissolve in water to form
comparatively weak acids. The element Y is :
(a) Sulphur (b) Nitrogen (c) Phosphrous (d) Carbon
Ans. (c)
Sol. P
15

Allotropic forms white P, red P, black P, P2


fertilizers : Ammonium phosphate
P4O6 + 6H2O ¾® 4H3PO3
P4O10 + 6H2O ¾® 4H3PO4
22. How many sigma bonds are present between any two carbon atoms in fullerenes?
(a) 1 (b) 2 (c) 3 (d) 4
Ans. (a)

6
Sol. Number of sigma bonds between any two atoms will be equal to one.
23. Four gram of mixture of calcium carbonte and sand is treated with excess of HCl and 0.880 g of carbon-di-oxide is
produced. What is the percentage of calcium carbonate in original mixture?
(a) 40% (b) 50% (c) 55% (d) 45%
Ans. (b)
Sol. CaCO3 + 2HCl ¾® CaCl2 + H2O + CO2

0.88
CO2 = moles = = 0.02 moles
44
CaCO3
amount = 0.02 × 100 = 2g

2
% of CaCO3 in the mixture is, × 100 = 50%
4
24. A student was studying reactions of metals with dilute NaOH at room temperature. The student took dilute NaOH
in four different test tubes and added Copper powder to test tube A, Zinc dust to test tube B, Aluminium powder to
test tube C and Iron powder to test tube D and observed effervescence in.
(a) Test tubes A & B (b) Test tubes B & C (c) Test tubes C & D (d) Test tubes A & D
Ans. (b)
Sol. For the given infomation,
Zn and Al can only react will aqeous NaOH because of their amphoteric nature.
Zn + 2NaOH ¾® Na2ZnO2 + H2 ­
Al + NaOH + H2O ¾® NaAlO2 + 1.5 H2 ­
25. A magician performed following act: He dipped Rs. 50 note in a 50% solution of alcohol in water and held it on the
burning flame, but the note did not burn. The reason behind this is-
(a) The alcohol kept on dousing the fire
(b) Air required for burning was not available
(c) The Rs 50 note failed to reach ignition temperature
(d) The Rs. 50 note is fire proof
Ans. (c)
Sol. Conceptual question
26. Gammaxene insecticide powder is prepared by the reaction given in the adjacent box. If 78 g of benzene when
reacted with 106.5 g of chlorine, how much Gammaxene would be formed?

(a) 140 g (b) 154.5 g (c) 145.5 g (d) 160 g


Ans. (c)

7
Sol. Cl H
H Cl
Cl H
+ 3 Cl2 UV
H Cl
Benzene H
Cl
H Cl
Gammaxene
78g 106.5 g (L.R.)

291
Amount of gammaxene formed is, × 106.5 = 145.5 g
3 ´ 71

27. Arrange following solutions in increasing hydronium ion concentration. The solutions are :
(P) 0.1 M HCl
(Q) 0.1 M H2SO4
(R) 0.001 M NH4OH
(S) 0.001 M Ca(OH)2
(a) P > Q > R > S (b) Q > P > S > R (c) S > R > Q > P (d) S > R > P > Q
Ans. (Bonus)
Sol. Correct option should be Q > P > R > S
28. A zinc rod was dipped in 100 cm3 of 1M copper chloride solution. After certain time the molarity of Cu 2+ ions in the
solution was found to be 0.8 M. If the weight of zinc rod, 20 g, then the molarity of chloride ions is ____.
(a) 2M (b)1.5M (c) 1 M (d) 0.5 M
Ans. (a)
Sol. CuCl2 (aq) ¾® Cu2+(aq) + 2Cl– (aq)
1M 1M 2M
the actual reaction is, Cu+2 + Zn ¾® Zn+2 + Cu.
(There will be no change in concentration of Cl– ions)
29. Which of the following polymeric material will be ideal for remolding?
(a) Polythene and Melamine (b) Polyvinyl chloride and Polythene
(c) Melamine and Bakelite (d) Bakelite and Polyvinyl chloride
Ans. (b)
Sol. Polyvinyl chloride and polyethene are thermoplastics.
30. When four dilute solutions of (I) vinegar, (II) common salt, (III) caustic soda and (IV) baking soda are tested with
universal indicator which will be the correct observation
(a) I- Green, II - Violet, III - Blue, IV - Red
(b) I - Green, II - Blue, III -Violet IV- Red
(c) I - Red, H - Green, III - Violet, IV - Blue
(d) I-Red, II- Violet, III - Green, IV - Blue
Ans. (c)
Sol. Vinegar – acidic, NaCl – Netural, NaOH - Strong Base, Baking Soda-weak base.

8
31. Substance X is white crystalline solid which melts after 10 seconds on burner flame. It is soluble in water and
insoluble in CCl4 It is a poor conductor of electricity in molten state as well as in the form of aqueous solution, hence
we conclude that substance X is
(a) an ionic compound (b) a non polar covalent compound
(c) a polar covalent compound (d) a pure element
Ans. (c)
Sol. Informative question.
32. In a beaker 50 ml of a normal HC1 solution was taken and NH3 gas was passed through it for some time. The
contents of the beaker were then titrated, which required 60 ml of semi normal NaOH solution. How much ammonia
was passed through the beaker?
(a) 0.85 g (b) 0.34 g (c)0.51 g (d) 0.4 g
Ans. (b)
Sol. HCl taken = 1 × 50 = 50 m.xmol

1
NaOH consumed = × 60 = 30 m.mol
2
NH3 Reacted = 20 m.mol.

20 ´ 17
Amount of NH3 passed is,
1000

W = 0.34 g
33. Which is the correct order of metals with reference to their melting point in increasing order?
(a) Hg, Ga, Li, Ca (b) Ca, Li, Ga, Hg (c) Hg, Li, Ga, Ca (d) Hg, Ga, Ca, Li
Ans. (a)

Sol. Element Melting point


Hg –38.83 °C
Ga 29.76 °C
Li 180.5 °C
Ca 842 °C
34. Sodium tungstate has formula Na 2WO4, lead phosphate has formula Pb3(PO4)2, formula for lead tungstate should
be:
(a) PbWO4 (b) Pb2(WO4)3 (c) Pb3(WO4)2 (d) Pb3(WO4)4
Ans. (a)
Sol. Na2WO4 ¾® WO42–, Na+
Pb2(PO4)2 ¾® Pb2+, PO43–
Hence, the compound will be Pb (WO4)
35. What is the ratio of reducing agent to oxidizing agent, if the following reaction is correctly balanced?
NH3 + O2 ¾® NO + H2O
(a) 4 : 5 (b) 5 : 4 (c) 5 : 3 (d) 3 : 5
Ans. (a)
Sol. Balanced Equation is
4 NH3 + 5O2 ¾® 4NO + 6H2O
Ratio of Reduing agent to Oxidizing agent is 4 : 5.
9
36. Which of the following is iso-structural with CO2?
(a) NO2 (b) N2O4 (c) NO (d) N2O
Ans. (d)
Sol. O = C = O
– +
N=N=O

37. In one litre of pure water, 44.4 g of calcium chloride is dissolved. The number of ions in one mL of the resultant
solution is :
(a) 7.23 × 1023 (b) 7.23 × 1020 (c) 4.82 × 1023 (d) 4.82 × 1020
Ans. (b)

44.4 1
Sol. Molarity = × = 0.4 molar
111 1
CaCl2(aq) ¾® Ca2+(aq) + 2Cl–(aq)
0.4 m mol 0.4 m mol 2 × 0.4 m mol
Ions = 1.2 × 10 × 6.023 × 10 = 7.23 × 1020
–3 23

38. Which of the following species is / are isoelectronic with Neon?


(i) N3– (ii) Mg2+ (iii) K+ (iv) Ca2+
(a) only (iv) (b) only (ii) (c) both (i) and (ii) (d) both (i) and (iii)
Ans. (c)

Sol. Species No. of electrons


Ne 10 e–
N 3– 10 e–
Mg 2+ 10 e–
39. Which of the following gases will have equal volume at STP, if the weight of gases is 14.0 g?
(i) N2O (ii) NO2 (iii) N2 (iv) CO
(a) (i) & (ii) (b) (ii) & (iii) (c) (i) & (iii) (d) (iii) & (iv)
Ans. (d)
Sol. For the given amount of gas (14g) at S.T.P. as volume is same, molecular mass should be same.
Molecular mass : N2 = 28 ; CO = 28
40. Which of the following are not ionic?
(i) AlCl3 (ii) CaCl2 (iii) MgCl2 (iv) LiCl
(a) (i) and (iv) (b) (i) and (ii) (c) (ii) and (iii) (d) (iii) and (iv)
Ans. (a, d)
Sol. AlCl3, MgCl2, LiCl are covalent in nature.
41. Apples dropping from apple trees were observed by many many people before Newton. But why they fall, was
explained by Isaac Newton postulating the law of universal gravitation. Which of the following was explained by
Isaac Newton postulating statements best describes the situation.
(a) The force of gravity acts only on the apple
(b) The apple is attracted towards the surface of the earth
(c) Both earth and apple experience the same force of attraction towards each other
(d) Apple falls due to earth's gravity and hence only (a) is true and (c) is absurd

10
Ans. (c)
Sol. Both earth and apple will attract each other with force

G m 1m 2
FG =
r2

42. A rectangular metal plate, shown in the adjacent figure has a charge of 420 mC assumed to be uniformly distributed
over it. Then how much is the charge over the shaded area? No part of metal plate is cut. (Circles and the diagonal
are shown for clarity only. p = 22/7)

14 cm
28 cm
(a) 45 µC (b) 450 µC (c) 15 µC (d) 150 µC
Ans. (a)

14 ´ 28 14 ´ 28 22 2
Sol. Area of shaded region = - pr 2 = - ´ 7 = 42 cm2
2 2 7

420
\ charge of shaded area = 42 × mC = 45 mC
14 ´ 28
43. A piece of wire P and three identical cells are connected in series. An amount of heat is generated in a certain time
interval in the wire due to passage of current. Now the circuit is modified by replacing P with another wire Q and N
identical cells, all connected in series. Q is four times longer in length than P. The wire P and Q are of same material
and have the same diameter. If the heat generated in second situation is also same as before in the same time
interval, then find N.
(a) 4 (b) 6 (c) 16 (d) 36
Ans. (b)
Sol. Given ;
Power in circuit - I = Power in circuit - II

11
9e2 N 2 e2 9 N2
Þ R = R Þ =
ëéQ l Q = 4 l P Þ R Q = 4R P ûù
P Q R P 4R P

Þ N2 = 36 Þ N=6

44. A piece of ice is floating in water at 4° C in a beaker. When the ice melts completely, the water level in the beaker will
(a) rise (b) fall (c) remains unchanged (d) unpredictable
Ans. (a)
Sol. Since ice at 0° melts completely in water at 4°C.
Equilibrium temperature will be slightly less than 4°C.
Density of water is maximum at 4°C.
\ Hence volume of water increases as ice melts completely. m
So water level will rise.
Mathematically
4°C <4°C
m
V1 = .... (1)
rw

m
V2 = .... (2)
r'w

Q r'w < rw
\ V2 > V1
Hence water level rises.
45. In the adjacent circuit, the voltages across AD, BD and CD are 2 V, 6 V and 8 V respectively. If resistance RA = 1
kW, then the values of resistances RB and RC are _____ and _____ respectively.
RB
(a) 4 kW and 6 kW B A
(b) 2 kW and 1 kW
RC RA
(c) 1 kW and 2 kW

(d) data insufficient as battery voltage is not given C D

Ans. (b)
Sol. Given; VAD = 2V
Þ i2 RA = 2 .... (1)
RB
Also, VBD = 6V B A
i i
Þ i (RA + RB) = 6
2
.... (2)
and, VCD = 8 RC RA

Þ i2(RA + RB + RC) = 8 .... (3) i D


C
Dividing (2) by (1)

R A + RB
=3 Þ RB = 2RA Þ R B = 2kW
RA

12
R A + RB + RC
Dividing (3) by (1) we get =4
RA

Þ 3RA + RC = 4RA Þ RC = RA Þ R C = 1kW

46. Some waveforms among I, II, III and IV superpose (add graphically) to produce the waveforms P, Q, R and S.
Among the following, match the pairs that give the correct combinations:

I0 P t
t
Q t

II0 t
R t

III0 t
IV0 t S
t

Resultant Superposition of
P (K) III and IV
Q (L) II and IV
R (M) I, II and III
S (N) I and IV
(O) II and III
(a) P « O, Q « N, R « L, S « M (b) P « M, Q « N, R « L, S « K
(c) P « M, Q « N, R « K, S « L (d) P « O, Q « M, R « L, S « K
Ans. (b)
Sol. By observation option (b) should be correct.
47. Refer to the adjacent figure. A variable force F is applied to a body of mass 6 kg at rest. The body moves along
x - axis as shown. The speed of the body at x = 5 m and x = 6 m is ____ and ____ respectively.

(a) 0 m/s, 0 m/s (b) 0 m/s, 2 m/s (c) 2 m/s, 2 m/s (d) 2 m/s, 4 m/s
Ans. (c)
Sol. area under curve given us the work done.
(area under curve from t = 0 to t = 5) = (area under curve from t = 0 to t = 6)

1
\ area = ´ 4 ´ 4 + 1 ´ 4 = 12 sq. units
2

\ work done = DK

13
1
Þ 12 = ´ 6 ´ v2 Þ v = 2m / s at both x = 5 m & x = 6 m
2

48. A rigid body of mass m is suspended from point O using an inextensible string of length L When it is displaced
through an angle q, what is the change in the potential energy of the mass? (Refer adjacent figure.)
X
O

Y
(a) mg L (1 – cosq) (b) mg L (cos q – 1) (c) mg L cos q (d) mg L (1 – sin q)
Ans. (a)

q
Lcosq L
Sol.

h = L – Lcosq

DU = mgh = mgL(1 – cosq)


49. Consider the motion of a small spherical steel body of mass m, falling freely through a long column of a fluid that
opposes its motion with a force proportional to its speed. Initially the body moves down fast, but after some time
attains a constant velocity known as terminal velocity. If weight mg, opposing force (Fv) and buoyant force (Fb) act
on the body, then the correct equation relating these forces, after the terminal velocity is reached, is:
(a) mg + Fv = Fb (b) mg = Fv – Fb (c) mg = Fv + Fb (d) None of these
Ans. (c)
Sol. To acquire terminal velocity
Net force on ball should be zero so

FV + FB

[mg =FV + FB]


mg

æ1 ö
50. At any instant of time, the total energy (E) of a simple pendulum is equal to the sum of its kinetic energy ç mv 2 ÷
è2 ø
æ 1 2ö
and potential energy ç kx ÷ , where, m is the mass, v is the velocity, x is the displacement of the bob and k is a
è2 ø
constant for the pendulum. The amplitude of oscillation of the pendulum is 10 cm and its total energy is 4 mJ.
Find k
(a) 1.8 Nm–1 (b) 0.8 Nm–1 (c) 0.5 Nm–1 (d) data insufficient
Ans. (b)

14
1 2
Sol. E= kA = 4 ´ 10-3 J
2

1
( )
2
´ k ´ 10 -1 = 4 ´ 10 -3
2

k = 8 ´ 10-1 = 0.8 Nm -1

51. A particle experiences constant acceleration for 20 s after starting from rest. If it travels a distance S1 in the first 10
s and distance S2 in the next 10 s, the relation between S1 and S2 is:
(a) S2 = 3S1 (b) S1= 3S2 (c) S2= 2S1 (d) S1=10S2
Ans. (a)

Sol.

1 1
S1 = a (10)2 = a 100 = 50a
2 2

1
S2 = [ a (20)2] – [50a] = 150a
2

S2 = 3S1

52. When a charged particle with charge q and mass m enters uniform magnetic field B with velocity v at right angles to
B the force on the moving particle is given by qvB. This force acts as the centripetal force making the charged

mv
particle go in a uniform circular motion with radius r = . Now if a hydrogen ion and a deuterium ion enter the
Bq
magnetic field with velocities in the ratio 2:1 respectively, then the ratio of their radii will be _____.
(a) 1 : 2 (b) 2 : 1 (c) 1 : 4 (d) 1 : 1
Ans. (d)

mv
Sol. r =
Bq

mH v H
For Hydrogen ion rH = Bq ..........(1)
H

md v d
For Deuterium ion rd = Bq ..........(2)
d

(1) rH mH v H q d 1 2 1 1
Þ ´ ´ ´
(2) rd = md v d q H = 2 1 1 = 1

15
53. A physics teacher and his family are travelling in a car on a highway during a severe lightning storm. Choose the
correct option:
(a) Safest place will be inside the car as the charges due to lightning tend to remain on the metal sheet / skin of the
vehicle if struck by lightning.
(b) It's too dangerous to be inside the car. As the car has a metal body the charges tend to accumulate on the
surface and will generate a strong electric field inside the car.
(c) Safest place is under a tree. It's better to get drenched under a tree as the wet tree will provide a path to the
charges for earthing.
(d) It is safer to exit the car and stand on open ground.
Ans. (a)
Sol. By the concept of electrostatic shielding.
54. The radius of curvature of a convex mirror is 'x'. The distance of an object from focus of this mirror is 'y'. Then what
is the distance of image from the focus?
(a) y2/4x (b) x2/y (c) x2/4y (d) 4y2/x
Ans. (c)
Sol. By Newton's formula
f2 = x1x2 where x1 ® Object distance from focus
x2 ® Image distance from focus.

2
æxö
ç 2 ÷ = yx2 x
è ø f=
2

x2
= yx2
4

x2
x2 =
4y

55. In a screw-nut assembly (shown below) the nut is held fixed in its position and the screw is allowed to roatate inside
it A convex lens (L) of focal length 6.0 cm is fixed on the nut. An object pin (P) is attached to the screw head. The
image of the object is observed on a screen Y. When the screw head is rotated through one rotation, the linear
distance moved by the screw tip is 1.0 mm. The observations are made only when the image is obtained in the same
orientation on the screen. At a certain position of P, the image formed is three times magnified as that of the pin
height. Through how many turns should the screw head be rotated so that the image is two times magnified?
L
Y P

O X

Screen nut
screw head
XI
(a) 8 (b) 10 (c) 12 (d) 14
16
Ans. (b)
Sol. For convex lense

f
m=
f+u

6
For Ist image ( -3 ) =
6 + u1

–18 –3u1 = 6
3u1 = –24
u1 = –8 c.m.

6
For IInd image ( –2) =
6 + u2

Þ u2 = –9 cm
|Du| = 1 c.m. = 10 mm

10mm
So no. of roatations = = 10 rotations
1mm

56. The triangular face of a crown glass prism ABC is isosceles. Length AB = length AC and the rectangular face with
edge AC is silvered. A ray of light is incident normally on rectangular face with edge AB. It undergoes reflections at
AC and AB internally and it emerges normally through the rectangular base with edge BC. Then angle BAC of the
prism is _____.
(a) 24° (b) 30° (c) 36° (d) 42°
Ans. (c)

A
Sol.
A

90°
A
A
A
–2
90

2A
–2A

2A
90

2A 2A
B C

since ÐB = ÐC
and by geometry ÐB = 2ÐA
so ÐA + ÐB + ÐC = 180°
A + 2A + 2A = 180°
5A = 180°
17
180°
A= = 36°
5

57. A sound wave is produced by a vibrating metallic string stretched between its ends. Four statements are given below.
Some of them are correct.
(P) Sound wave is produced inside the string.
(Q) Sound wave in the string is transverse.
(R) Wavelength of the sound wave in surrounding air is equal to the wavelength of the transverse wave on the string.
(S) Loudness of sound is proportional to the square of the amplitude of the vibrating string.
Choose the correct option.
(a) P (b) R and S (c) P and Q (d) S
Ans. (d)
Sol. Since sound wave produced by the string will be longitudinal in nature.
The loudness of sound depends on the amplitude of vibration of the vibrating object.
58. A conductor in the form of a circular loop is carrying current I. The direction of the current is as shown. Then which
figure represents the correct direction of magnetic field lines on the surfaces of the planes XY and XZ. (Consider
those surfaces of the XY and XZ planes which are seen in the figure.)

Z Z

I I

0 0
(a) (b)
Y Y

X X

Z Z

I I

0 0
(c) (d)
Y Y

X X

Ans. (a)
Sol. By right hand thumb rule option (a) is correct.
59. A school is located between two cliffs. When the metal bell is struck by school attendant, first echo is heard by him
after 2.4 s and second echo follows after 2.0 s for him at the same position near the bell. If the velocity of sound in
air is 340 ms–1 at the temperature of the surroundings, then the distance between the cliffs is approximately _____.
(a) 0.488 km (b) 0.751 km (c) 1.16 km (d) 1.41 km
Ans. (c)

Sol.
d1
d2
IInd Ist
D
18
For echo from right sided cliff

2d1 v ´ t1 340 ´ 2.4


t1 = Þ d1 = =
v 2 2

= 170 × 2.4
= 408m
For echo from left cliff

2d 2 v ´ t 2 340 ´ ( 4.4 )
t2 = Þ d2 = =
v 2 2

= 170 × 4.4
= 748 m
D = d1 + d2 = 408 + 748 = 1156 m
= 1.156 m
= 1.16 k.m.
60. A new linear scale of temperature measurement is to be designed. It is called a 'Z scale' on which the freezing and
boiling points of water are 20 Z and 220 Z respectively. What will be the temperature shown on the 'Z scale'
corresponding to a temperature of 20° C on the Celsius scale?
(a) 10 Z (b) 20 Z (c) 40 Z (d) 60 Z
Ans. (d)

C-0 Z - 20
Sol. =
100 220 - 20

C Z - 20
=
100 200
z = 20 + 2C = 20 + 2×(20) = 60

3a 6 + a 8
61. Let a and b the roots of x2 – 5x + 3 = 0 with a > b. If an = an – bn for n ³ 1 the the value of a7
is

(a) 2 (b) 3 (c) 4 (d) 5


Ans. (d)
Sol. x2 – 5x + 3 = 0
Þ x2 + 3 = 5x
then, a2 + 3 = 5a
Þ a8 + 3x6 = 5a7 ......(1)
and b8 + 3b6 = 5b7 ......(2)
Subtracting equation (2) from equation (1)
3(a6 – b6) + a8 – b8 = 5(a7 – b7)

3a 6 + a8
Þ =5
a7

19
62. In the given figure, two concentric circles are shown with centre O. PQRS is a square inscribed in the outer circle. It
also circumscribes the inner circle, touching it at points B, C, D and A. What is the ratio of the perimeter of the outer
circle to that of quadrilateral ABCD?

P B Q

A O C

S D R

p 3p p
(a) (b) (c) (d) p
4 2 2

Ans. (c)

Sol. Let radius of inner circle be r, then AB = 2r


Þ AC = 2r = PQ = QR

so, PR = PQ2 + QR 2 = 2 PQ = 2 2r
PR
Þ OR = = 2r
2
Required Ratio is 2p( 2 r) : 4 2 r
Þ p:2
63. In rectangle ABCD, AB=5 and BC=3. Points F and G are on the line segment CD so that DF = 1 and GC=2.
Lines AF and BG intersect at E. What is the area of DAEB?
(a) 10 sq. units (b) 15/2 sq. units (c) 25/2 sq. units (d) 20 sq. units
Ans. (c)
Sol. DEFG ~ DEAB E

EM 2
Þ =
EN 5 D 1 F 2 G 2 C
M
EN - MN 2
Þ =
EN 5

3 2 3
Þ 1- =
EN 5
Þ EN = 5
1 25 A N B
ar (DABE) = ×5×5= sq. units 5
2 2
64. The number of triples (x,y,z) such that any one of these numbers is added to the product of the other two, the result
is 2, is
(a) 1 (b) 2 (c) 4 (d) infinitely many
Ans. (b)
Sol. xy + z = 2 .....(1)
yz + x = 2 .....(2)
zx + y = 2 .....(3)
20
equation (1) – equation (2)
Þ y(x – z) + z – x = 0
Þ (x – z) (y – 1) = 0
Either x = z or y = 1
Case-1 x = z
Þ x(y + 1) = 2 .....(4) (from equation 1)
Þ x2 + y = 2 Þ x2 = 2 – y .....(5) (from equation 3)

x2 (y + 1)2 4
and equation (4)/(5) Þ =
x 2
2-y
Þ (y2 + 2y + 1) (z – y) = y
y2 – 3y + 2 = 0
Þ (y – 1)2 (y + 2) = 0
When y = 1 we get x = z = 1
and when y = –2 we get x = z = –2
Case-2 y = 1
x+z=2 (from equation 1)
xz = 1 (from equation 3)
On solving these equation we get x = z = 1
Therefore possible triples are
(1, 1, 1) & (–2, –2, –2)

65. What is the product of all the roots of the equation 5|x | +8 = x 2 – 16 ?

(a) – 64 (b) – 24 (c) 576 (d) 24


Ans. (a)

Sol. 5 x + 8 = x 2 - 16

Þ 5|x| + 8 = x2 –16
Þ |x|2 – 5|x| – 24 = 0
Þ (|x| – 8) (|x| + 3) = 0
Þ |x| = 8 As |x| ¹ –3
Þ x=±8
\ Products of the roots = –64
66. How many positive integers N give a remainder 8 when 2008 is divided by N.
(a) 12 (b) 13 (c)14 (d) 15
Ans. (d)
Sol. Given : 2008 º 8 (mod N)
Þ 2008 = NK + 8, K Î N
Þ NK = 24 × 53
\ Number of factors = 20
Leaving factors 1, 2, 4, 5, 8 as divisor > remainder
So number of required positive integers = 20 – 5 = 15

21
67. LCM of two numbers is 5775. Which of the following cannot be their HCF?
(a) 175 (b) 231 (c) 385 (d) 455
Ans. (d)
Sol. Let a and b be the two number
The, LCM (a, b) = 5775 = 52 × 3 × 7 × 11
Here, possible products are 231, 175, 385
\ 455 is not possible.

1 1 1
68. If a, b, c are distinct real numbers such that a + = b + = c + evaluate abc.
b c a

(a) ± 2 (b) 2 –1 (c) 3 (d) ±1

Ans. (d)

1 1 1
Sol. a+ =b+ =c+
b c a

1 1 b-c
Then, a – b = – =
c b bc

1 1 c -a
b–c= - =
a c ac

1 1 a-b
c–a= - =
b a ab

(a - b)(b - c)(c - a)
Þ (a – b)(b – c)(c – a) =
a 2 b 2c 2

é 1 ù
Þ (a – b)(b – c)(c – a) ê1 - =0
ë a b2c 2 úû
2

1
So, either (a – b)(b – c)(c – a) = 0 or 1 - =0
( abc )2
But a = b, b = c, c = a is not possible as a, b, c are distinct.
So, a2b2c2 = 1
\ abc = ± 1

69. Mr. X with his eight children of different ages is on a family trip. His oldest child, who is 9 years old saw a license
plate with a 4-digit number in which each of two digits appear two times. "Look daddy" she exclaims. "That number
is evenly divisble by the age of each of us kids! "That's right," replies Mr. X, "and the last two digits just happen to be
my age" Which of the following is not the age of one of Mr. X's children?
(a) 4 (b) 5 (c) 6 (d) 7
Ans. (b)

22
Sol. age of oldest child = 9 years
\ age of 7 younger childs Î {1, 2, ...., 8}
The 4 digit number is divisible by all the ages of 8 childrens. And in {1, 2, ..., 9}, there are 4 even numbers. So
atleast 3 childs will be even aged, so that 4 digit number will be surely divisible by all even number so option (a) 4
and (c) 6 will be neglected.
Let's check option (b) i.e., 5.
If none of children has age 5, then smallest number divisible by rest of the numbers is 9 × 8 × 7 = 504 and checking
its multiples we get 504 × 11 = 5544 a four digit number with same two digits.
So, age of X = 44
age of children = 1, 2, 3, 4, 6, 7, 8, 9.
So answer is option (b) 5.
70. How many numbers lie between 11 and 1111 which divided by 9 leave a remainder 6 and when divided by 21 leave
a remainder 12?
(a) 18 (b) 28 (c) 8 (d) None of these
Ans. (a)
Sol. 33 is one of the number that satisfies the condition
Required number will be k × LCM (21, 9) + 33
Þ 63k + 33
Þ 11 < 63k + 33 < 1111

-22 1078
<k<
63 63

k = 0, 1, 2, ........ 17
18 numbers
71. If the equation (a2 – 5a + 6)x2 + (a2 – 3a + 2) x + (a2 – 4) = 0 has more than two roots, then the value of a is
(a) 2 (b) 3 (c) 1 (d) None of these
Ans. (a)
Sol. As the quadratic equation has more than 2 roots, so it is an identity.
\ a2 – 5a + 6 = 0
(a – 2) (a – 3) = 0
Þ a = 2, a = 3
a2 – 3a + 2 = 0
(a – 2 (a + 2) = 0
Þ a = 2, a = 1
a2 – 4 = 0
(a – 2) (a + 2) = 0
Þ a = 2, a = – 2
The common value in all three equations is a = 2
\ a=2

23
1 1 1
72. The sum of the roots of + = is zero. The product of roots is
x+a x+b c

a+b 1 2 2
(a) 0 (b) (c) – (a + b ) (d) 2(a2+b2)
2 2

Ans. (c)
1 1 1
Sol. + =
x+a x+b c
1 1 1
Þ = –
x+a c x+b
1 x+b-c
Þ =
x + a c(x + b)
(x + a) (x + b – c) = (cx + bc)
x2 + bx – cx + ax + ab – ac = (cx + bc)
x2 + (b + a – 2c) x + ab – ac – bc = 0
Sum of roots = 0
\ b + a = 2c
Product of roots
Þ ab – (a + b) c

æa + bö
ab – (a + b) ç ÷
è 2 ø
2ab - a 2 - b 2 - 2ab –1 2
2
Þ
2
(
a + b2 )
73. The solution of the equation 1 + 4 + 7+.....+ x = 925 is
(a) 73 (b) 76 (c) 70 (d) 74
Ans. (a)
Sol. 1 + 4 + 7+.....+ x = 925
a = 1, d = 3

n
[2(1) + (n – 1)3] = 925
2
n [2 + 3n – 3] = 1850
n [3n – 1] = 1850
3n2 – n – 1850 = 0
3n2 – 75n + 74n – 1850 = 0
3n(n – 25) + 74(n – 25) = 0
n = 25
\ x = T25 = 1 + (25 – 1)3 = 1 + 72 = 73
74. Two unbiased dice are rolled. What is the probability of getting a sum which is neither 7 nor 11?
(a) 7/9 (b) 7/18 (c) 2/9 (d) 11/18
Ans. (a)

24
Sol. For sum = 7, Favourable cases = (1, 6); (2, 5); (3, 4); (4, 3); (5, 2); (6, 1)
For sum = 11, Favourable cases = (6, 5); (5, 6)
Total events = 36
Let A = event of getting sum 7, B = event of getting sum 11.

6 1 2 1
P(A) = = P(B) = =
36 6 36 18

1 1 4 2
P(A È B) = + = =
6 18 18 9

2 7
P( A È B ) = 1 – = .
9 9

75. An observer standing at the top of a tower, find that the angle of elevation of a red bulb on the top of a light house
of height H is a. Further, he finds that the angle of depression of reflection of the bulb in the ocean is b. Therefore,
the height of the tower is

H(tan b – tan a) H sin(b – a) H(cos a – cos b)


(a) (b) (c) (d) H
(tan b + tan a) cos(a + b) (cot a + cot b)

Ans. (a)

H-h
Sol. tan a = .....(1)
x
H–h
H+h
tan b = .....(2) a H
x x
b
h
tan a H - h
Equation (1)/(2) Þ =
tan b H + h
H+h H
tan a + tan b 2H
Þ = (Using componendo and dividendo)
tan a - tan b -2h

æ tan b - tan a ö
Þ h = Hç ÷
è tan b + tan a ø

76. In the convex quadrilateral ABCD, the diagonals AC and BD meet at O and the measure of angle AOB is 30°. If the
areas of triangle AOB, BOC COD and AOD are 1, 2, 8 and 4 square units respectively, what is the product of the
lengths of the diagonals AC and DB in sq. units?
(a) 60 (b) 56 (c) 54 (d) 64
Ans. (a)

AO ar( DAOD) 1
Sol. = =
OC ar( DDOC) 2

25
Þ AC = AO + OC = 3AO B

BO ar(DBOC) 2 1 A
Also = = = 1
OD ar( DCOD) 8 4
30°
Þ BD = BO + OD = 5 BO 2
4 O
1
ar(DAOB) = AO × BO × sin30° = 1 8
2

Þ AO × BO = 4 D C
Þ AC × BD = 3AO × 5BO = 3 × 5 × 4 = 60
77. If tan q + sec q = 1.5, then value of sin q is

5 12 3 2
(a) (b) (c) (d)
13 13 5 3
Ans. (a)

3
Sol. Given secq + tanq = 1.5 = and sec2q – tan2q = 1
2
Þ (secq – tanq) (secq + tanq) = 1

2
= secq – tanq =
3

13
\ secq =
12

5
So, sin q =
13

78. If sin2x + sin2y + sin2z = 0, then which of the following is NOT a possible value of cos x + cosy + cosz ?
(a) 3 (b) – 3 (c) – 1 (d) – 2
Ans. (d)
Sol. Given sin2x + sin2y + sin2z = 0
Þ sin x = sin y = sin z = 0
\ x, y, z Î np where n Î z
So, possible values of cos x + cos y + cos z = 3, – 3, –1, 1
Hence –2 is not possible.
79. Find the remainder when x51 is divided by x2 – 3x + 2.
(a) x (b) (251 – 2) x + 2 – 251 (c) (251 – 1)x + 2 – 251 (d) 0
Ans. (c)
Sol. Let the remainder be Ax + B
Then, By Remainder Theorem,
x51 = (x2 – 3x + 2) Q(x) + Ax + B
When x = 1, A + B = 1 ......(1)
and when x = 2, 2A + B = 2 51
......(2)
Solving (1) and (2)
A = 251 – 1 and B = 2 – 251
\ Remainder is (251 – 1)x + (2 – 251)
26
80. In an equilateral triangle, three coins of radii 1 unit each are kept so that they touch each other and also sides of the
triangle. The area of triangle ABC (in sq. units) is

7 3 7 3
(a) 4 + 2 3 (b) 4 3 + 6 (c) 12 + (d) 3 +
4 4

Ans. (b)
Sol. Here, PQ = QR = PR = 2

Then, PM = 3
As G is the centroid of DABC and DPQR A

1 1 1
\ GM = 3 ( 3 ) = and GN = 1 +
3 3
P
1
Let x se the side of DABC then, AN = GN
3

Q M R
1 æ 3x ö 3 +1
Þ ç ÷=
3è 2 ø 3 B C
N

Þ x = 2(1 + 3)

3 2
So, Area of DABC = (x )
4

3
= 4(1 + 3)2
4

= 3(4 + 2 3)

=6+ 4 3
Alternate Solution

QL 1 1
= tan 30° Þ = Þ BL =
BL BL 3 3 A

Similarly CM = 3
and LM = QR = 2 P

BC = 3 + 2 + 3 = 2(1 + 3)

3 Q R
So, Area of DABC = (BC2 )
4
30° 30°
3 B C
= 4(1 + 3)2 L M
4
= 3(4 + 2 3) = 6 + 4 3

27

You might also like